Anda di halaman 1dari 132

The Physics GRE Solution Guide

GR9677 Test

FT
RA
http://groups.yahoo.com/group/physicsgre_v2

November 3, 2009
D

Author:
David S. Latchman
2

FT
RA
D

David S. Latchman ©2009


Preface

This solution guide initially started out on the Yahoo Groups web site and was pretty

FT
successful at the time. Unfortunately, the group was lost and with it, much of the the
hard work that was put into it. This is my attempt to recreate the solution guide and
make it more widely avaialble to everyone. If you see any errors, think certain things
could be expressed more clearly, or would like to make suggestions, please feel free to
do so.
David Latchman
RA
Document Changes
05-11-2009 1. Added diagrams to GR0177 test questions 1-25
2. Revised solutions to GR0177 questions 1-25

04-15-2009 First Version


D
ii

FT
RA
D

David S. Latchman ©2009


Contents

FT
Preface i

Classical Mechanics xv
0.1 Kinematics . . . . . . . . . . . . . . . . . . . . . . . . . . . . . . . . . . . . xv
0.2 Newton’s Laws . . . . . . . . . . . . . . . . . . . . . . . . . . . . . . . . . xvi
0.3 Work & Energy . . . . . . . . . . . . . . . . . . . . . . . . . . . . . . . . . xvii
RA
0.4 Oscillatory Motion . . . . . . . . . . . . . . . . . . . . . . . . . . . . . . . xviii
0.5 Rotational Motion about a Fixed Axis . . . . . . . . . . . . . . . . . . . . xxii
0.6 Dynamics of Systems of Particles . . . . . . . . . . . . . . . . . . . . . . . xxiv
0.7 Central Forces and Celestial Mechanics . . . . . . . . . . . . . . . . . . . xxiv
0.8 Three Dimensional Particle Dynamics . . . . . . . . . . . . . . . . . . . . xxvi
0.9 Fluid Dynamics . . . . . . . . . . . . . . . . . . . . . . . . . . . . . . . . . xxvi
0.10 Non-inertial Reference Frames . . . . . . . . . . . . . . . . . . . . . . . . xxvii
D

0.11 Hamiltonian and Lagrangian Formalism . . . . . . . . . . . . . . . . . . . xxvii

Electromagnetism xxix
0.12 Electrostatics . . . . . . . . . . . . . . . . . . . . . . . . . . . . . . . . . . . xxix
0.13 Currents and DC Circuits . . . . . . . . . . . . . . . . . . . . . . . . . . . xxxiv
0.14 Magnetic Fields in Free Space . . . . . . . . . . . . . . . . . . . . . . . . . xxxiv
0.15 Lorentz Force . . . . . . . . . . . . . . . . . . . . . . . . . . . . . . . . . . xxxiv
0.16 Induction . . . . . . . . . . . . . . . . . . . . . . . . . . . . . . . . . . . . . xxxiv
0.17 Maxwell’s Equations and their Applications . . . . . . . . . . . . . . . . . xxxiv
0.18 Electromagnetic Waves . . . . . . . . . . . . . . . . . . . . . . . . . . . . . xxxiv
iv Contents
0.19 AC Circuits . . . . . . . . . . . . . . . . . . . . . . . . . . . . . . . . . . . xxxiv
0.20 Magnetic and Electric Fields in Matter . . . . . . . . . . . . . . . . . . . . xxxiv
0.21 Capacitance . . . . . . . . . . . . . . . . . . . . . . . . . . . . . . . . . . . xxxv
0.22 Energy in a Capacitor . . . . . . . . . . . . . . . . . . . . . . . . . . . . . . xxxv
0.23 Energy in an Electric Field . . . . . . . . . . . . . . . . . . . . . . . . . . . xxxv
0.24 Current . . . . . . . . . . . . . . . . . . . . . . . . . . . . . . . . . . . . . . xxxv
0.25 Current Destiny . . . . . . . . . . . . . . . . . . . . . . . . . . . . . . . . . xxxv
0.26 Current Density of Moving Charges . . . . . . . . . . . . . . . . . . . . . xxxv
0.27 Resistance and Ohm’s Law . . . . . . . . . . . . . . . . . . . . . . . . . . xxxv
0.28 Resistivity and Conductivity . . . . . . . . . . . . . . . . . . . . . . . . . . xxxvi

FT
0.29 Power . . . . . . . . . . . . . . . . . . . . . . . . . . . . . . . . . . . . . . . xxxvi
0.30 Kirchoff’s Loop Rules . . . . . . . . . . . . . . . . . . . . . . . . . . . . . . xxxvi
0.31 Kirchoff’s Junction Rule . . . . . . . . . . . . . . . . . . . . . . . . . . . . xxxvi
0.32 RC Circuits . . . . . . . . . . . . . . . . . . . . . . . . . . . . . . . . . . . . xxxvi
0.33 Maxwell’s Equations . . . . . . . . . . . . . . . . . . . . . . . . . . . . . . xxxvi
RA
0.34 Speed of Propagation of a Light Wave . . . . . . . . . . . . . . . . . . . . xxxvii
0.35 Relationship between E and B Fields . . . . . . . . . . . . . . . . . . . . . xxxvii
0.36 Energy Density of an EM wave . . . . . . . . . . . . . . . . . . . . . . . . xxxviii
0.37 Poynting’s Vector . . . . . . . . . . . . . . . . . . . . . . . . . . . . . . . . xxxviii

Optics & Wave Phonomena xxxix


0.38 Wave Properties . . . . . . . . . . . . . . . . . . . . . . . . . . . . . . . . . xxxix
0.39 Superposition . . . . . . . . . . . . . . . . . . . . . . . . . . . . . . . . . . xxxix
D

0.40 Interference . . . . . . . . . . . . . . . . . . . . . . . . . . . . . . . . . . . xxxix


0.41 Diffraction . . . . . . . . . . . . . . . . . . . . . . . . . . . . . . . . . . . . xxxix
0.42 Geometrical Optics . . . . . . . . . . . . . . . . . . . . . . . . . . . . . . . xxxix
0.43 Polarization . . . . . . . . . . . . . . . . . . . . . . . . . . . . . . . . . . . xxxix
0.44 Doppler Effect . . . . . . . . . . . . . . . . . . . . . . . . . . . . . . . . . . xl
0.45 Snell’s Law . . . . . . . . . . . . . . . . . . . . . . . . . . . . . . . . . . . . xl

Thermodynamics & Statistical Mechanics xli


0.46 Laws of Thermodynamics . . . . . . . . . . . . . . . . . . . . . . . . . . . xli
0.47 Thermodynamic Processes . . . . . . . . . . . . . . . . . . . . . . . . . . . xli

David S. Latchman ©2009


Contents v
0.48 Equations of State . . . . . . . . . . . . . . . . . . . . . . . . . . . . . . . . xli
0.49 Ideal Gases . . . . . . . . . . . . . . . . . . . . . . . . . . . . . . . . . . . . xli
0.50 Kinetic Theory . . . . . . . . . . . . . . . . . . . . . . . . . . . . . . . . . . xli
0.51 Ensembles . . . . . . . . . . . . . . . . . . . . . . . . . . . . . . . . . . . . xli
0.52 Statistical Concepts and Calculation of Thermodynamic Properties . . . xlii
0.53 Thermal Expansion & Heat Transfer . . . . . . . . . . . . . . . . . . . . . xlii
0.54 Heat Capacity . . . . . . . . . . . . . . . . . . . . . . . . . . . . . . . . . . xlii
0.55 Specific Heat Capacity . . . . . . . . . . . . . . . . . . . . . . . . . . . . . xlii
0.56 Heat and Work . . . . . . . . . . . . . . . . . . . . . . . . . . . . . . . . . xlii
0.57 First Law of Thermodynamics . . . . . . . . . . . . . . . . . . . . . . . . . xlii

FT
0.58 Work done by Ideal Gas at Constant Temperature . . . . . . . . . . . . . xliii
0.59 Heat Conduction Equation . . . . . . . . . . . . . . . . . . . . . . . . . . . xliii
0.60 Ideal Gas Law . . . . . . . . . . . . . . . . . . . . . . . . . . . . . . . . . . xliv
0.61 Stefan-Boltzmann’s FormulaStefan-Boltzmann’s Equation . . . . . . . . xliv
0.62 RMS Speed of an Ideal Gas . . . . . . . . . . . . . . . . . . . . . . . . . . xliv
RA
0.63 Translational Kinetic Energy . . . . . . . . . . . . . . . . . . . . . . . . . . xliv
0.64 Internal Energy of a Monatomic gas . . . . . . . . . . . . . . . . . . . . . xliv
0.65 Molar Specific Heat at Constant Volume . . . . . . . . . . . . . . . . . . . xlv
0.66 Molar Specific Heat at Constant Pressure . . . . . . . . . . . . . . . . . . xlv
0.67 Equipartition of Energy . . . . . . . . . . . . . . . . . . . . . . . . . . . . xlv
0.68 Adiabatic Expansion of an Ideal Gas . . . . . . . . . . . . . . . . . . . . . xlvii
0.69 Second Law of Thermodynamics . . . . . . . . . . . . . . . . . . . . . . . xlvii
D

Quantum Mechanics xlix


0.70 Fundamental Concepts . . . . . . . . . . . . . . . . . . . . . . . . . . . . . xlix
0.71 Schrödinger Equation . . . . . . . . . . . . . . . . . . . . . . . . . . . . . . xlix
0.72 Spin . . . . . . . . . . . . . . . . . . . . . . . . . . . . . . . . . . . . . . . . liv
0.73 Angular Momentum . . . . . . . . . . . . . . . . . . . . . . . . . . . . . . liv
0.74 Wave Funtion Symmetry . . . . . . . . . . . . . . . . . . . . . . . . . . . . liv
0.75 Elementary Perturbation Theory . . . . . . . . . . . . . . . . . . . . . . . liv

Atomic Physics lv
0.76 Properties of Electrons . . . . . . . . . . . . . . . . . . . . . . . . . . . . . lv

©2009 David S. Latchman


vi Contents
0.77 Bohr Model . . . . . . . . . . . . . . . . . . . . . . . . . . . . . . . . . . . lv
0.78 Energy Quantization . . . . . . . . . . . . . . . . . . . . . . . . . . . . . . lvi
0.79 Atomic Structure . . . . . . . . . . . . . . . . . . . . . . . . . . . . . . . . lvi
0.80 Atomic Spectra . . . . . . . . . . . . . . . . . . . . . . . . . . . . . . . . . lvi
0.81 Selection Rules . . . . . . . . . . . . . . . . . . . . . . . . . . . . . . . . . . lvii
0.82 Black Body Radiation . . . . . . . . . . . . . . . . . . . . . . . . . . . . . . lvii
0.83 X-Rays . . . . . . . . . . . . . . . . . . . . . . . . . . . . . . . . . . . . . . lviii
0.84 Atoms in Electric and Magnetic Fields . . . . . . . . . . . . . . . . . . . . lix

Special Relativity lxiii

FT
0.85 Introductory Concepts . . . . . . . . . . . . . . . . . . . . . . . . . . . . . lxiii
0.86 Time Dilation . . . . . . . . . . . . . . . . . . . . . . . . . . . . . . . . . . lxiii
0.87 Length Contraction . . . . . . . . . . . . . . . . . . . . . . . . . . . . . . . lxiii
0.88 Simultaneity . . . . . . . . . . . . . . . . . . . . . . . . . . . . . . . . . . . lxiii
0.89 Energy and Momentum . . . . . . . . . . . . . . . . . . . . . . . . . . . . lxiv
0.90 Four-Vectors and Lorentz Transformation . . . . . . . . . . . . . . . . . . lxv
RA
0.91 Velocity Addition . . . . . . . . . . . . . . . . . . . . . . . . . . . . . . . . lxvi
0.92 Relativistic Doppler Formula . . . . . . . . . . . . . . . . . . . . . . . . . lxvi
0.93 Lorentz Transformations . . . . . . . . . . . . . . . . . . . . . . . . . . . . lxvi
0.94 Space-Time Interval . . . . . . . . . . . . . . . . . . . . . . . . . . . . . . . lxvii

Laboratory Methods lxix


0.95 Data and Error Analysis . . . . . . . . . . . . . . . . . . . . . . . . . . . . lxix
D

0.96 Instrumentation . . . . . . . . . . . . . . . . . . . . . . . . . . . . . . . . . lxxi


0.97 Radiation Detection . . . . . . . . . . . . . . . . . . . . . . . . . . . . . . . lxxi
0.98 Counting Statistics . . . . . . . . . . . . . . . . . . . . . . . . . . . . . . . lxxi
0.99 Interaction of Charged Particles with Matter . . . . . . . . . . . . . . . . lxxii
0.100Lasers and Optical Interferometers . . . . . . . . . . . . . . . . . . . . . . lxxii
0.101Dimensional Analysis . . . . . . . . . . . . . . . . . . . . . . . . . . . . . lxxii
0.102Fundamental Applications of Probability and Statistics . . . . . . . . . . lxxii

GR9677 Exam Solutions lxxiii


0.103Discharge of a Capacitor . . . . . . . . . . . . . . . . . . . . . . . . . . . . lxxiii

David S. Latchman ©2009


Contents vii
0.104Magnetic Fields & Induced EMFs . . . . . . . . . . . . . . . . . . . . . . . lxxiii
0.105A Charged Ring I . . . . . . . . . . . . . . . . . . . . . . . . . . . . . . . . lxxiv
0.106A Charged Ring II . . . . . . . . . . . . . . . . . . . . . . . . . . . . . . . . lxxiv
0.107Forces on a Car’s Tires . . . . . . . . . . . . . . . . . . . . . . . . . . . . . lxxv
0.108Block sliding down a rough inclined plane . . . . . . . . . . . . . . . . . lxxv
0.109Collision of Suspended Blocks . . . . . . . . . . . . . . . . . . . . . . . . . lxxvi
0.110Damped Harmonic Motion . . . . . . . . . . . . . . . . . . . . . . . . . . lxxvii
0.111Spectrum of the Hydrogen Atom . . . . . . . . . . . . . . . . . . . . . . . lxxvii
0.112Internal Conversion . . . . . . . . . . . . . . . . . . . . . . . . . . . . . . . lxxviii
0.113The Stern-Gerlach Experiment . . . . . . . . . . . . . . . . . . . . . . . . . lxxviii

FT
0.114Positronium Ground State Energy . . . . . . . . . . . . . . . . . . . . . . lxxviii
0.115Specific Heat Capacity and Heat Lost . . . . . . . . . . . . . . . . . . . . . lxxix
0.116Conservation of Heat . . . . . . . . . . . . . . . . . . . . . . . . . . . . . . lxxix
0.117Thermal Cycles . . . . . . . . . . . . . . . . . . . . . . . . . . . . . . . . . lxxix
0.118Mean Free Path . . . . . . . . . . . . . . . . . . . . . . . . . . . . . . . . . lxxx
RA
0.119Probability . . . . . . . . . . . . . . . . . . . . . . . . . . . . . . . . . . . . lxxxi
0.120Barrier Tunneling . . . . . . . . . . . . . . . . . . . . . . . . . . . . . . . . lxxxii
0.121Distance of Closest Appraoch . . . . . . . . . . . . . . . . . . . . . . . . . lxxxii
0.122Collisions and the He atom . . . . . . . . . . . . . . . . . . . . . . . . . . lxxxiii
0.123Oscillating Hoops . . . . . . . . . . . . . . . . . . . . . . . . . . . . . . . . lxxxiii
0.124Mars Surface Orbit . . . . . . . . . . . . . . . . . . . . . . . . . . . . . . . lxxxiv
0.125The Inverse Square Law . . . . . . . . . . . . . . . . . . . . . . . . . . . . lxxxiv
D

0.126Charge Distribution . . . . . . . . . . . . . . . . . . . . . . . . . . . . . . . lxxxv


0.127Capacitors in Parallel . . . . . . . . . . . . . . . . . . . . . . . . . . . . . . lxxxvi
0.128Resonant frequency of a RLC Circuit . . . . . . . . . . . . . . . . . . . . . lxxxvi
0.129Graphs and Data Analysis . . . . . . . . . . . . . . . . . . . . . . . . . . . lxxxvii
0.130Superposition of Waves . . . . . . . . . . . . . . . . . . . . . . . . . . . . lxxxviii
0.131The Plank Length . . . . . . . . . . . . . . . . . . . . . . . . . . . . . . . . lxxxix
0.132The Open Ended U-tube . . . . . . . . . . . . . . . . . . . . . . . . . . . . xc
0.133Sphere falling through a viscous liquid . . . . . . . . . . . . . . . . . . . . xc
0.134Moment of Inertia and Angular Velocity . . . . . . . . . . . . . . . . . . . xci
0.135Quantum Angular Momentum . . . . . . . . . . . . . . . . . . . . . . . . xcii

©2009 David S. Latchman


viii Contents
0.136Invariance Violations and the Non-conservation of Parity . . . . . . . . . xcii
0.137Wave function of Identical Fermions . . . . . . . . . . . . . . . . . . . . . xciii
0.138Relativistic Collisions . . . . . . . . . . . . . . . . . . . . . . . . . . . . . . xciii
0.139Relativistic Addition of Velocities . . . . . . . . . . . . . . . . . . . . . . . xciii
0.140Relativistic Energy and Momentum . . . . . . . . . . . . . . . . . . . . . xciv
0.141Ionization Potential . . . . . . . . . . . . . . . . . . . . . . . . . . . . . . . xciv
0.142Photon Emission and a Singly Ionized He atom . . . . . . . . . . . . . . . xcv
0.143Selection Rules . . . . . . . . . . . . . . . . . . . . . . . . . . . . . . . . . . xcvi
0.144Photoelectric Effect . . . . . . . . . . . . . . . . . . . . . . . . . . . . . . . xcvi
0.145Stoke’s Theorem . . . . . . . . . . . . . . . . . . . . . . . . . . . . . . . . . xcvii

FT
0.1461-D Motion . . . . . . . . . . . . . . . . . . . . . . . . . . . . . . . . . . . . xcvii
0.147High Pass Filter . . . . . . . . . . . . . . . . . . . . . . . . . . . . . . . . . xcvii
0.148Generators and Faraday’s Law . . . . . . . . . . . . . . . . . . . . . . . . xcviii
0.149Faraday’s Law and a Wire wound about a Rotating Cylinder . . . . . . . xcviii
0.150Speed of π+ mesons in a laboratory . . . . . . . . . . . . . . . . . . . . . . xcix
RA
0.151Transformation of Electric Field . . . . . . . . . . . . . . . . . . . . . . . . xcix
0.152The Space-Time Interval . . . . . . . . . . . . . . . . . . . . . . . . . . . . xcix
0.153Wavefunction of the Particle in an Infinte Well . . . . . . . . . . . . . . . c
0.154Spherical Harmonics of the Wave Function . . . . . . . . . . . . . . . . . c
0.155Decay of the Positronium Atom . . . . . . . . . . . . . . . . . . . . . . . . c
0.156Polarized Electromagnetic Waves I . . . . . . . . . . . . . . . . . . . . . . c
0.157Polarized Electromagnetic Waves II . . . . . . . . . . . . . . . . . . . . . . ci
D

0.158Total Internal Reflection . . . . . . . . . . . . . . . . . . . . . . . . . . . . ci


0.159Single Slit Diffraction . . . . . . . . . . . . . . . . . . . . . . . . . . . . . . ci
0.160The Optical Telescope . . . . . . . . . . . . . . . . . . . . . . . . . . . . . . cii
0.161Pulsed Lasers . . . . . . . . . . . . . . . . . . . . . . . . . . . . . . . . . . cii
0.162Relativistic Doppler Shift . . . . . . . . . . . . . . . . . . . . . . . . . . . . ciii
0.163Gauss’ Law, the Electric Field and Uneven Charge Distribution . . . . . civ
0.164Capacitors in Parallel . . . . . . . . . . . . . . . . . . . . . . . . . . . . . . cv
0.165Standard Model . . . . . . . . . . . . . . . . . . . . . . . . . . . . . . . . . cv
0.166Nuclear Binding Energy . . . . . . . . . . . . . . . . . . . . . . . . . . . . cv
0.167Work done by a man jumping off a boat . . . . . . . . . . . . . . . . . . . cvi

David S. Latchman ©2009


Contents ix
0.168Orbits and Gravitational Potential . . . . . . . . . . . . . . . . . . . . . . cvi
0.169Schwartzchild Radius . . . . . . . . . . . . . . . . . . . . . . . . . . . . . . cvi
0.170Lagrangian of a Bead on a Rod . . . . . . . . . . . . . . . . . . . . . . . . cvii
0.171Ampere’s Law . . . . . . . . . . . . . . . . . . . . . . . . . . . . . . . . . . cvii
0.172Larmor Formula . . . . . . . . . . . . . . . . . . . . . . . . . . . . . . . . . cviii
0.173The Oscilloscope and Electron Deflection . . . . . . . . . . . . . . . . . . cix
0.174Negative Feedback . . . . . . . . . . . . . . . . . . . . . . . . . . . . . . . cix
0.175Adiabatic Work of an Ideal Gas . . . . . . . . . . . . . . . . . . . . . . . . cx
0.176Change in Entrophy of Two Bodies . . . . . . . . . . . . . . . . . . . . . . cx
0.177Double Pane Windows . . . . . . . . . . . . . . . . . . . . . . . . . . . . . cxi

FT
0.178Gaussian Wave Packets . . . . . . . . . . . . . . . . . . . . . . . . . . . . . cxii
0.179Angular Momentum Spin Operators . . . . . . . . . . . . . . . . . . . . . cxii
0.180Semiconductors and Impurity Atoms . . . . . . . . . . . . . . . . . . . . cxii
0.181Specific Heat of an Ideal Diatomic Gas . . . . . . . . . . . . . . . . . . . . cxii
0.182Transmission of a Wave . . . . . . . . . . . . . . . . . . . . . . . . . . . . . cxiii
RA
0.183Piano Tuning & Beats . . . . . . . . . . . . . . . . . . . . . . . . . . . . . . cxiii
0.184Thin Films . . . . . . . . . . . . . . . . . . . . . . . . . . . . . . . . . . . . cxiv
0.185Mass moving on rippled surface . . . . . . . . . . . . . . . . . . . . . . . cxv
0.186Normal Modes and Couples Oscillators . . . . . . . . . . . . . . . . . . . cxv
0.187Waves . . . . . . . . . . . . . . . . . . . . . . . . . . . . . . . . . . . . . . . cxv
0.188Charged Particles in E&M Fields . . . . . . . . . . . . . . . . . . . . . . . cxvi
0.189Rotation of Charged Pith Balls in a Collapsing Magnetic Field . . . . . . cxvi
D

0.190Coaxial Cable . . . . . . . . . . . . . . . . . . . . . . . . . . . . . . . . . . cxvii


0.191Charged Particles in E&M Fields . . . . . . . . . . . . . . . . . . . . . . . cxviii
0.192THIS ITEM WAS NOT SCORED . . . . . . . . . . . . . . . . . . . . . . . cxix
0.193The Second Law of Thermodynamics . . . . . . . . . . . . . . . . . . . . cxix
0.194Small Oscillations . . . . . . . . . . . . . . . . . . . . . . . . . . . . . . . . cxix
0.195Period of Mass in Potential . . . . . . . . . . . . . . . . . . . . . . . . . . . cxx
0.196Internal Energy . . . . . . . . . . . . . . . . . . . . . . . . . . . . . . . . . cxxi
0.197Specific Heat of a Super Conductor . . . . . . . . . . . . . . . . . . . . . . cxxi
0.198Pair Production . . . . . . . . . . . . . . . . . . . . . . . . . . . . . . . . . cxxi
0.199Probability Current Density . . . . . . . . . . . . . . . . . . . . . . . . . . cxxii

©2009 David S. Latchman


x Contents
0.200Quantum Harmonic Oscillator Energy Levels . . . . . . . . . . . . . . . . cxxii
0.201Three Level LASER and Metastable States . . . . . . . . . . . . . . . . . . cxxiii
0.202Quantum Oscillator – Raising and Lowering Operators . . . . . . . . . . cxxiv

Constants & Important Equations cxxv


.1 Constants . . . . . . . . . . . . . . . . . . . . . . . . . . . . . . . . . . . . . cxxv
.2 Vector Identities . . . . . . . . . . . . . . . . . . . . . . . . . . . . . . . . . cxxv
.3 Commutators . . . . . . . . . . . . . . . . . . . . . . . . . . . . . . . . . . cxxvi
.4 Linear Algebra . . . . . . . . . . . . . . . . . . . . . . . . . . . . . . . . . . cxxvii

FT
RA
D

David S. Latchman ©2009


List of Tables

FT
0.67.1Table of Molar Specific Heats . . . . . . . . . . . . . . . . . . . . . . . . . xlvi

0.119.1
Table of wavefunction amplitudes . . . . . . . . . . . . . . . . . . . . . . lxxxii
0.181.1
Table of degrees of freedom of a Diatomic atom . . . . . . . . . . . . . . . cxiii

.1.1 Something . . . . . . . . . . . . . . . . . . . . . . . . . . . . . . . . . . . . cxxv


RA
D
xii List of Tables

FT
RA
D

David S. Latchman ©2009


List of Figures

FT
0.201.1
Three Level Laser . . . . . . . . . . . . . . . . . . . . . . . . . . . . . . . . cxxiv
RA
D
xiv List of Figures

FT
RA
D

David S. Latchman ©2009


Classical Mechanics

0.1 Kinematics

FT
0.1.1 Linear Motion

Average Velocity

∆x x2 − x1
v= = (0.1.1)
∆t t2 − t1
RA
Instantaneous Velocity

∆x dx
v = lim = = v(t) (0.1.2)
∆t→0 ∆t dt

Kinematic Equations of Motion

The basic kinematic equations of motion under constant acceleration, a, are


D

v = v0 + at (0.1.3)
v2 = v20 + 2a (x − x0 ) (0.1.4)
1
x − x0 = v0 t + at2 (0.1.5)
2
1
x − x0 = (v + v0 ) t (0.1.6)
2

0.1.2 Circular Motion

In the case of Uniform Circular Motion, for a particle to move in a circular path, a
radial acceleration must be applied. This acceleration is known as the Centripetal
Acceleration
xvi Classical Mechanics
Centripetal Acceleration

v2
a= (0.1.7)
r

Angular Velocity

v
ω= (0.1.8)
r
We can write eq. (0.1.7) in terms of ω

a = ω2 r (0.1.9)

FT
Rotational Equations of Motion

The equations of motion under a constant angular acceleration, α, are

ω = ω0 + αt (0.1.10)
ω + ω0
θ= t (0.1.11)
2
1
θ = ω0 t + αt2 (0.1.12)
RA
2
ω = ω0 + 2αθ
2 2
(0.1.13)

0.2 Newton’s Laws

0.2.1 Newton’s Laws of Motion


First Law A body continues in its state of rest or of uniform motion unless acted upon
by an external unbalanced force.
D

Second Law The net force on a body is proportional to its rate of change of momentum.
dp
F= = ma (0.2.1)
dt

Third Law When a particle A exerts a force on another particle B, B simultaneously


exerts a force on A with the same magnitude in the opposite direction.

FAB = −FBA (0.2.2)

0.2.2 Momentum

p = mv (0.2.3)

David S. Latchman ©2009


Work & Energy xvii
0.2.3 Impulse
w
∆p = J = Fdt = Favg dt (0.2.4)

0.3 Work & Energy

0.3.1 Kinetic Energy

1
K ≡ mv2 (0.3.1)
2

FT
0.3.2 The Work-Energy Theorem

The net Work done is given by


Wnet = K f − Ki (0.3.2)

0.3.3 Work done under a constant Force


RA
The work done by a force can be expressed as

W = F∆x (0.3.3)

In three dimensions, this becomes

W = F · ∆r = F∆r cos θ (0.3.4)

For a non-constant force, we have

wx f
D

W= F(x)dx (0.3.5)
xi

0.3.4 Potential Energy

The Potential Energy is


dU(x)
F(x) = − (0.3.6)
dx
for conservative forces, the potential energy is

wx
U(x) = U0 − F(x0 )dx0 (0.3.7)
x0

©2009 David S. Latchman


xviii Classical Mechanics
0.3.5 Hooke’s Law

F = −kx (0.3.8)
where k is the spring constant.

0.3.6 Potential Energy of a Spring

1
U(x) = kx2 (0.3.9)
2

0.4 Oscillatory Motion

0.4.1 Equation for Simple Harmonic Motion

FT
x(t) = A sin (ωt + δ) (0.4.1)
where the Amplitude, A, measures the displacement from equilibrium, the phase, δ, is
the angle by which the motion is shifted from equilibrium at t = 0.
RA
0.4.2 Period of Simple Harmonic Motion


T= (0.4.2)
ω

0.4.3 Total Energy of an Oscillating System


D

Given that
x = A sin (ωt + δ) (0.4.3)
and that the Total Energy of a System is

E = KE + PE (0.4.4)

The Kinetic Energy is

1
KE = mv2
2
1 dx
= m
2 dt
1
= mA2 ω2 cos2 (ωt + δ) (0.4.5)
2

David S. Latchman ©2009


Oscillatory Motion xix
The Potential Energy is
1
U = kx2
2
1
= kA2 sin2 (ωt + δ) (0.4.6)
2
Adding eq. (0.4.5) and eq. (0.4.6) gives
1
E = kA2 (0.4.7)
2

0.4.4 Damped Harmonic Motion


dx
Fd = −bv = −b (0.4.8)

FT
dt
where b is the damping coefficient. The equation of motion for a damped oscillating
system becomes
dx d2 x
− kx − b = m 2 (0.4.9)
dt dt
Solving eq. (0.4.9) goves
x = Ae−αt sin (ω0 t + δ) (0.4.10)
We find that
RA
b
α= (0.4.11)
2m
r
k b2
ω0 = −
m 4m2
r
b2
= ω20 −
4m2
q
= ω20 − α2 (0.4.12)
D

0.4.5 Small Oscillations

The Energy of a system is


1
E = K + V(x) = mv(x)2 + V(x) (0.4.13)
2
We can solve for v(x), r
2
v(x) = (E − V(x)) (0.4.14)
m
where E ≥ V(x) Let the particle move in the potential valley, x1 ≤ x ≤ x2 , the potential
can be approximated by the Taylor Expansion
" # " 2 #
dV(x) 1 2 d V(x)
V(x) = V(xe ) + (x − xe ) + (x − xe ) + ··· (0.4.15)
dx x=xe 2 dx2 x=xe

©2009 David S. Latchman


xx Classical Mechanics
At the points of inflection, the derivative dV/dx is zero and d V/dx2 is positive. This
2

means that the potential energy for small oscillations becomes

1
V(x) u V(xe ) + k(x − xe )2 (0.4.16)
2
where " #
d2 V(x)
k≡ ≥0 (0.4.17)
dx2 x=xe

As V(xe ) is constant, it has no consequences to physical motion and can be dropped.


We see that eq. (0.4.16) is that of simple harmonic motion.

0.4.6 Coupled Harmonic Oscillators

FT
Consider the case of a simple pendulum of length, `, and the mass of the bob is m1 .
For small displacements, the equation of motion is

θ̈ + ω0 θ = 0

We can express this in cartesian coordinates, x and y, where


(0.4.18)
RA
x = ` cos θ ≈ ` (0.4.19)
y = ` sin θ ≈ `θ (0.4.20)

eq. (0.4.18) becomes


ÿ + ω0 y = 0 (0.4.21)
This is the equivalent to the mass-spring system where the spring constant is
mg
k = mω20 = (0.4.22)
`
D

This allows us to to create an equivalent three spring system to our coupled pendulum
system. The equations of motion can be derived from the Lagrangian, where

L=T−V
1 2 1 2 1 2 1 2 1 2
 
= m ẏ1 + m ẏ2 − ky1 + κ y2 − y1 + ky2
2 2 2 2 2
1  2  1   2 
= m y˙1 + y˙2 2 − k y21 + y22 + κ y2 − y1 (0.4.23)
2 2
We can find the equations of motion of our system

d ∂L ∂L
!
= (0.4.24)
dt ∂ ẏn ∂yn
1
Add figure with coupled pendulum-spring system

David S. Latchman ©2009


Oscillatory Motion xxi
The equations of motion are
m ÿ1 = −ky1 + κ y2 − y1

(0.4.25)
m ÿ2 = −ky2 + κ y2 − y1

(0.4.26)
We assume solutions for the equations of motion to be of the form
y1 = cos(ωt + δ1 ) y2 = B cos(ωt + δ2 )
(0.4.27)
ÿ1 = −ωy1 ÿ2 = −ωy2
Substituting the values for ÿ1 and ÿ2 into the equations of motion yields
 
k + κ − mω2 y1 − κy2 = 0 (0.4.28)
 
−κy1 + k + κ − mω2 y2 = 0 (0.4.29)
We can get solutions from solving the determinant of the matrix

FT
k + κ − mω2

−κ  = 0 (0.4.30)
−κ k + κ − mω2
Solving the determinant gives
 2  
mω2 − 2mω2 (k + κ) + k2 + 2kκ = 0 (0.4.31)
This yields
g

k
=


`

ω2 =  m
RA

(0.4.32)

 k + 2κ g 2κ
= +


m ` m

We can now determine exactly how the masses move with each mode by substituting
ω2 into the equations of motion. Where
k
ω2 = We see that
m
k + κ − mω2 = κ (0.4.33)
Substituting this into the equation of motion yields
y1 = y2 (0.4.34)
D

We see that the masses move in phase with each other. You will also notice
the absense of the spring constant term, κ, for the connecting spring. As the
masses are moving in step, the spring isn’t stretching or compressing and hence
its absence in our result.
k+κ
ω2 = We see that
m
k + κ − mω2 = −κ (0.4.35)
Substituting this into the equation of motion yields
y1 = −y2 (0.4.36)
Here the masses move out of phase with each other. In this case we see the
presence of the spring constant, κ, which is expected as the spring playes a role.
It is being stretched and compressed as our masses oscillate.

©2009 David S. Latchman


xxii Classical Mechanics
0.4.7 Doppler Effect

The Doppler Effect is the shift in frequency and wavelength of waves that results from
a source moving with respect to the medium, a receiver moving with respect to the
medium or a moving medium.

Moving Source If a source is moving towards an observer, then in one period, τ0 , it


moves a distance of vs τ0 = vs / f0 . The wavelength is decreased by
vs v − vs
λ0 = λ − − (0.4.37)
f0 f0
The frequency change is
v v
 
f0 = = f0 (0.4.38)
λ0 v − vs

FT
Moving Observer As the observer moves, he will measure the same wavelength, λ, as
if at rest but will see the wave crests pass by more quickly. The observer measures
a modified wave speed.
v0 = v + |vr | (0.4.39)
The modified frequency becomes
v0 vr
 
f0 = = f0 1 + (0.4.40)
λ
RA
v

Moving Source and Moving Observer We can combine the above two equations
v − vs
λ0 = (0.4.41)
f0
v = v − vr
0
(0.4.42)

To give a modified frequency of


v0 v − vr
 
f = 0 =
0
f0 (0.4.43)
λ v − vs
D

0.5 Rotational Motion about a Fixed Axis

0.5.1 Moment of Inertia


Z
I= R2 dm (0.5.1)

0.5.2 Rotational Kinetic Energy


1
K = Iω2 (0.5.2)
2

David S. Latchman ©2009


Rotational Motion about a Fixed Axis xxiii
0.5.3 Parallel Axis Theorem

I = Icm + Md2 (0.5.3)

0.5.4 Torque

τ=r×F (0.5.4)
τ = Iα (0.5.5)

where α is the angular acceleration.

0.5.5 Angular Momentum

we can find the Torque FT


L = Iω

dL
(0.5.6)
RA
τ= (0.5.7)
dt

0.5.6 Kinetic Energy in Rolling

With respect to the point of contact, the motion of the wheel is a rotation about the
point of contact. Thus
1
K = Krot = Icontact ω2 (0.5.8)
2
D

Icontact can be found from the Parallel Axis Theorem.

Icontact = Icm + MR2 (0.5.9)

Substitute eq. (0.5.8) and we have

1 
K= Icm + MR2 ω2
2
1 1
= Icm ω2 + mv2 (0.5.10)
2 2

The kinetic energy of an object rolling without slipping is the sum of hte kinetic energy
of rotation about its center of mass and the kinetic energy of the linear motion of the
object.

©2009 David S. Latchman


xxiv Classical Mechanics
0.6 Dynamics of Systems of Particles

0.6.1 Center of Mass of a System of Particles

Position Vector of a System of Particles

m1 r1 + m2 r2 + m3 r3 + · · · + mN rN
R= (0.6.1)
M

Velocity Vector of a System of Particles

dR
V=

FT
dt
m1 v1 + m2 v2 + m3 v3 + · · · + mN vN
= (0.6.2)
M

Acceleration Vector of a System of Particles

dV
A=
dt
RA
m1 a1 + m2 a2 + m3 a3 + · · · + mN aN
= (0.6.3)
M

0.7 Central Forces and Celestial Mechanics

0.7.1 Newton’s Law of Universal Gravitation


GMm
 
F=− r̂ (0.7.1)
r2
D

0.7.2 Potential Energy of a Gravitational Force


GMm
U(r) = − (0.7.2)
r

0.7.3 Escape Speed and Orbits

The energy of an orbiting body is


E=T+U
1 GMm
= mv2 − (0.7.3)
2 r

David S. Latchman ©2009


Central Forces and Celestial Mechanics xxv
The escape speed becomes
1 GMm
E = mv2esc − =0 (0.7.4)
2 RE
Solving for vesc we find
r
2GM
vesc = (0.7.5)
Re

0.7.4 Kepler’s Laws

First Law The orbit of every planet is an ellipse with the sun at a focus.

Second Law A line joining a planet and the sun sweeps out equal areas during equal

FT
intervals of time.

Third Law The square of the orbital period of a planet is directly proportional to the
cube of the semi-major axis of its orbit.

T2
=C (0.7.6)
R3
where C is a constant whose value is the same for all planets.
RA
0.7.5 Types of Orbits

The Energy of an Orbiting Body is defined in eq. (0.7.3), we can classify orbits by their
eccentricities.

Circular Orbit A circular orbit occurs when there is an eccentricity of 0 and the orbital
energy is less than 0. Thus
D

1 2 GM
v − =E<0 (0.7.7)
2 r
The Orbital Velocity is
r
GM
v= (0.7.8)
r

Elliptic Orbit An elliptic orbit occurs when the eccentricity is between 0 and 1 but the
specific energy is negative, so the object remains bound.
r
2 1
 
v= GM − (0.7.9)
r a

where a is the semi-major axis

©2009 David S. Latchman


xxvi Classical Mechanics
Parabolic Orbit A Parabolic Orbit occurs when the eccentricity is equal to 1 and the
orbital velocity is the escape velocity. This orbit is not bounded. Thus

1 2 GM
v − =E=0 (0.7.10)
2 r
The Orbital Velocity is
r
2GM
v = vesc = (0.7.11)
r

Hyperbolic Orbit In the Hyperbolic Orbit, the eccentricity is greater than 1 with an
orbital velocity in excess of the escape velocity. This orbit is also not bounded.
r
GM
v∞ =

FT
(0.7.12)
a

0.7.6 Derivation of Vis-viva Equation

The total energy of a satellite is

1 GMm
E = mv2 − (0.7.13)
RA
2 r
For an elliptical or circular orbit, the specific energy is

GMm
E=− (0.7.14)
2a
Equating we get
2 1
 
v = GM −
2
(0.7.15)
r a
D

0.8 Three Dimensional Particle Dynamics

0.9 Fluid Dynamics

When an object is fully or partially immersed, the buoyant force is equal to the weight
of fluid displaced.

0.9.1 Equation of Continuity

ρ1 v1 A1 = ρ2 v2 A2 (0.9.1)

David S. Latchman ©2009


Non-inertial Reference Frames xxvii
0.9.2 Bernoulli’s Equation
1
P + ρv2 + ρgh = a constant (0.9.2)
2

0.10 Non-inertial Reference Frames

0.11 Hamiltonian and Lagrangian Formalism

0.11.1 Lagrange’s Function (L)

L=T−V (0.11.1)

FT
where T is the Kinetic Energy and V is the Potential Energy in terms of Generalized
Coordinates.

0.11.2 Equations of Motion(Euler-Lagrange Equation)

∂L d ∂L
!
= (0.11.2)
RA
∂q dt ∂q̇

0.11.3 Hamiltonian

H =T+V
= pq̇ − L(q, q̇) (0.11.3)

where
D

∂H
= q̇ (0.11.4)
∂p
∂H ∂L
=−
∂q ∂x
= −ṗ (0.11.5)

©2009 David S. Latchman


xxviii Classical Mechanics

FT
RA
D

David S. Latchman ©2009


Electromagnetism

0.12 Electrostatics

0.12.1 Coulomb’s Law

F12 =
1

FT
The force between two charged particles, q1 and q2 is defined by Coulomb’s Law.

q1 q2
4π0 r212

where 0 is the permitivitty of free space, where


!
r̂12

0 = 8.85 × 10−12 C2 N.m2


(0.12.1)

(0.12.2)
RA
0.12.2 Electric Field of a point charge

The electric field is defined by mesuring the magnitide and direction of an electric
force, F, acting on a test charge, q0 .
F
E≡ (0.12.3)
q0
The Electric Field of a point charge, q is
D

1 q
E= r̂ (0.12.4)
4π0 r2
In the case of multiple point charges, qi , the electric field becomes
n
1 X qi
E(r) = r̂i (0.12.5)
4π0 i=1 r2i

Electric Fields and Continuous Charge Distributions

If a source is distributed continuously along a region of space, eq. (0.12.5) becomes


Z
1 1
E(r) = r̂dq (0.12.6)
4π0 r2
xxx Electromagnetism
If the charge was distributed along a line with linear charge density, λ,

dq
λ= (0.12.7)
dx

The Electric Field of a line charge becomes

λ
Z
1
E(r) = r̂dx (0.12.8)
4π0 r2
line

In the case where the charge is distributed along a surface, the surface charge density
is, σ
Q dq
σ= = (0.12.9)

FT
A dA
The electric field along the surface becomes

σ
Z
1
E(r) = r̂dA (0.12.10)
4π0 r2
Surface

In the case where the charge is distributed throughout a volume, V, the volume charge
RA
density is
Q dq
ρ= = (0.12.11)
V dV
The Electric Field is
ρ
Z
1
E(r) = r̂dV (0.12.12)
4π0 r2
Volume
D

0.12.3 Gauss’ Law

The electric field through a surface is


I I
Φ= dΦ = E · dA (0.12.13)
surface S surface S

The electric flux through a closed surface encloses a net charge.


I
Q
E · dA = (0.12.14)
0

where Q is the charge enclosed by our surface.

David S. Latchman ©2009


Electrostatics xxxi
0.12.4 Equivalence of Coulomb’s Law and Gauss’ Law

The total flux through a sphere is


I
q
E · dA = E(4πr2 ) = (0.12.15)
0
From the above, we see that the electric field is
q
E= (0.12.16)
4π0 r2

0.12.5 Electric Field due to a line of charge

Consider an infinite rod of constant charge density, λ. The flux through a Gaussian

FT
cylinder enclosing the line of charge is
Z Z Z
Φ= E · dA + E · dA + E · dA (0.12.17)
top surface bottom surface side surface

At the top and bottom surfaces, the electric field is perpendicular to the area vector, so
for the top and bottom surfaces,
E · dA = 0 (0.12.18)
RA
At the side, the electric field is parallel to the area vector, thus

E · dA = EdA (0.12.19)

Thus the flux becomes, Z Z


Φ= E · dA = E dA (0.12.20)
side sirface

The area in this case is the surface area of the side of the cylinder, 2πrh.
D

Φ = 2πrhE (0.12.21)

Applying Gauss’ Law, we see that Φ = q/0 . The electric field becomes

λ
E= (0.12.22)
2π0 r

0.12.6 Electric Field in a Solid Non-Conducting Sphere

Within our non-conducting sphere or radius, R, we will assume that the total charge,
Q is evenly distributed throughout the sphere’s volume. So the charge density of our
sphere is
Q Q
ρ= = 4 (0.12.23)
V 3
πR 3

©2009 David S. Latchman


xxxii Electromagnetism
The Electric Field due to a charge Q is
Q
E= (0.12.24)
4π0 r2
As the charge is evenly distributed throughout the sphere’s volume we can say that
the charge density is
dq = ρdV (0.12.25)
where dV = 4πr2 dr. We can use this to determine the field inside the sphere by
summing the effect of infinitesimally thin spherical shells
Z E Z r
dq
E= dE = 2
0 0 4πr
ρ
Z r
= dr
0 0

FT
Qr
= 4 (0.12.26)
3
π 0 R3

0.12.7 Electric Potential Energy


1
U(r) = qq0 r (0.12.27)
4π0
RA
0.12.8 Electric Potential of a Point Charge

The electrical potential is the potential energy per unit charge that is associated with a
static electrical field. It can be expressed thus
U(r) = qV(r) (0.12.28)
And we can see that
1 q
V(r) = (0.12.29)
D

4π0 r
A more proper definition that includes the electric field, E would be
Z
V(r) = − E · d` (0.12.30)
C

where C is any path, starting at a chosen point of zero potential to our desired point.
The difference between two potentials can be expressed such
Z b Z a
V(b) − V(a) = − E · d` + E · d`
Z b
=− E · d` (0.12.31)
a

David S. Latchman ©2009


Electrostatics xxxiii
This can be further expressed
Z b
V(b) − V(a) = (∇V) · d` (0.12.32)
a

And we can show that


E = −∇V (0.12.33)

0.12.9 Electric Potential due to a line charge along axis

Let us consider a rod of length, `, with linear charge density, λ. The Electrical Potential
due to a continuous distribution is

FT
Z Z
1 dq
V= dV = (0.12.34)
4π0 r

The charge density is


dq = λdx (0.12.35)
Substituting this into the above equation, we get the electrical potential at some distance
x along the rod’s axis, with the origin at the start of the rod.
RA
1 dq
dV =
4π0 x
1 λdx
= (0.12.36)
4π0 x
This becomes
λ x2
 
V= ln (0.12.37)
4π0 x1
where x1 and x2 are the distances from O, the end of the rod.
Now consider that we are some distance, y, from the axis of the rod of length, `. We
D

again look at eq. (0.12.34), where r is the distance of the point P from the rod’s axis.
Z
1 dq
V=
4π0 r
Z `
1 λdx
=
4π0 0 x2 + y2  12
λ
   12 `
= ln x + x2 + y2
4π0 0
λ  12 
 
= ln ` + `2 + y2 − ln y
4π0
 1 
λ  ` + `2 + y2 2 
= ln   (0.12.38)
4π0  d


©2009 David S. Latchman


xxxiv Electromagnetism
0.13 Currents and DC Circuits

0.14 Magnetic Fields in Free Space

0.15 Lorentz Force

FT
4

0.16 Induction

5
RA
0.17 Maxwell’s Equations and their Applications

0.18 Electromagnetic Waves


D

0.19 AC Circuits

0.20 Magnetic and Electric Fields in Matter

David S. Latchman ©2009


Capacitance xxxv
0.21 Capacitance
Q = CV (0.21.1)

0.22 Energy in a Capacitor

Q2
U=
2C
CV 2
=
2
QV

FT
= (0.22.1)
2

0.23 Energy in an Electric Field

U 0 E2
u≡ = (0.23.1)
volume 2
RA
0.24 Current
dQ
I≡ (0.24.1)
dt

0.25 Current Destiny


D

Z
I= J · dA (0.25.1)
A

0.26 Current Density of Moving Charges


I
J= = ne qvd (0.26.1)
A

0.27 Resistance and Ohm’s Law


V
R≡ (0.27.1)
I

©2009 David S. Latchman


xxxvi Electromagnetism
0.28 Resistivity and Conductivity

L
R=ρ (0.28.1)
A

E = ρJ (0.28.2)

J = σE (0.28.3)

0.29 Power

0.30 Kirchoff’s Loop Rules

Write Here
FT
P = VI (0.29.1)
RA
0.31 Kirchoff’s Junction Rule

Write Here

0.32 RC Circuits
D

Q
E − IR − =0 (0.32.1)
C

0.33 Maxwell’s Equations

0.33.1 Integral Form

Gauss’ Law for Electric Fields


w Q
E · dA = (0.33.1)
0
closed surface

David S. Latchman ©2009


Speed of Propagation of a Light Wave xxxvii
Gauss’ Law for Magnetic Fields
w
B · dA = 0 (0.33.2)
closed surface

Ampère’s Law
z d w
B · ds = µ0 I + µ0 0 E · dA (0.33.3)
dt
surface

Faraday’s Law
z d w
E · ds = − B · dA (0.33.4)
dt
surface

0.33.2 Differential Form


Gauss’ Law for Electric Fields

Gauss’ Law for Magnetism

Ampère’s Law
FT
∇·E=

∇·B=0
ρ
0
(0.33.5)

(0.33.6)
RA
∂E
∇ × B = µ0 J + µ0 0 (0.33.7)
∂t
Faraday’s Law
∂B
∇·E=− (0.33.8)
∂t

0.34 Speed of Propagation of a Light Wave


1
D

c= √ (0.34.1)
µ0 0
In a material with dielectric constant, κ,
√ c
c κ = (0.34.2)
n
where n is the refractive index.

0.35 Relationship between E and B Fields

E = cB (0.35.1)
E·B=0 (0.35.2)

©2009 David S. Latchman


xxxviii Electromagnetism
0.36 Energy Density of an EM wave
!
1 B2
u= + 0 E2 (0.36.1)
2 µ0

0.37 Poynting’s Vector


1
S= E×B (0.37.1)
µ0

FT
RA
D

David S. Latchman ©2009


Optics & Wave Phonomena

0.38 Wave Properties

0.39

2
Superposition

FT
RA
0.40 Interference

0.41 Diffraction
D

0.42 Geometrical Optics

0.43 Polarization

6
xl Optics & Wave Phonomena
0.44 Doppler Effect
7

0.45 Snell’s Law

0.45.1 Snell’s Law

n1 sin θ1 = n2 sin θ2 (0.45.1)

0.45.2 Critical Angle and Snell’s Law

FT
The critical angle, θc , for the boundary seperating two optical media is the smallest
angle of incidence, in the medium of greater index, for which light is totally refelected.
From eq. (0.45.1), θ1 = 90 and θ2 = θc and n2 > n1 .

n1 sin 90 = n2 sinθc
sin θc =
n1
(0.45.2)
RA
n2
D

David S. Latchman ©2009


Thermodynamics & Statistical Mechanics

0.46 Laws of Thermodynamics

0.47

2 FT
Thermodynamic Processes
RA
0.48 Equations of State

0.49 Ideal Gases


D

0.50 Kinetic Theory

0.51 Ensembles

6
xlii Thermodynamics & Statistical Mechanics
0.52 Statistical Concepts and Calculation of Thermody-
namic Properties

0.53 Thermal Expansion & Heat Transfer

FT
0.54 Heat Capacity
 
Q = C T f − Ti (0.54.1)

where C is the Heat Capacity and T f and Ti are the final and initial temperatures
respectively.
RA
0.55 Specific Heat Capacity
 
Q = cm T f − ti (0.55.1)

where c is the specific heat capacity and m is the mass.


D

0.56 Heat and Work


Z Vf
W= PdV (0.56.1)
Vi

0.57 First Law of Thermodynamics

dEint = dQ − dW (0.57.1)

where dEint is the internal energy of the system, dQ is the Energy added to the system
and dW is the work done by the system.

David S. Latchman ©2009


Work done by Ideal Gas at Constant Temperature xliii
0.57.1 Special Cases to the First Law of Thermodynamics
Adiabatic Process During an adiabatic process, the system is insulated such that there
is no heat transfer between the system and its environment. Thus dQ = 0, so

∆Eint = −W (0.57.2)

If work is done on the system, negative W, then there is an increase in its internal
energy. Conversely, if work is done by the system, positive W, there is a decrease
in the internal energy of the system.

Constant Volume (Isochoric) Process If the volume is held constant, then the system
can do no work, δW = 0, thus
∆Eint = Q (0.57.3)

FT
If heat is added to the system, the temperature increases. Conversely, if heat is
removed from the system the temperature decreases.

Closed Cycle In this situation, after certain interchanges of heat and work, the system
comes back to its initial state. So ∆Eint remains the same, thus

∆Q = ∆W (0.57.4)

The work done by the system is equal to the heat or energy put into it.
RA
Free Expansion In this process, no work is done on or by the system. Thus ∆Q =
∆W = 0,
∆Eint = 0 (0.57.5)

0.58 Work done by Ideal Gas at Constant Temperature


Starting with eq. (0.56.1), we substitute the Ideal gas Law, eq. (0.60.1), to get
D

Z Vf
dV
W = nRT
Vi V
Vf
= nRT ln (0.58.1)
Vi

0.59 Heat Conduction Equation


The rate of heat transferred, H, is given by

Q TH − TC
H= = kA (0.59.1)
t L
where k is the thermal conductivity.

©2009 David S. Latchman


xliv Thermodynamics & Statistical Mechanics
0.60 Ideal Gas Law
PV = nRT (0.60.1)
where
n = Number of moles
P = Pressure
V = Volume
T = Temperature
and R is the Universal Gas Constant, such that
R ≈ 8.314 J/mol. K

FT
We can rewrite the Ideal gas Law to say
PV = NkT (0.60.2)
where k is the Boltzmann’s Constant, such that
R
k= ≈ 1.381 × 10−23 J/K
NA
RA
0.61 Stefan-Boltzmann’s FormulaStefan-Boltzmann’s Equa-
tion
P(T) = σT4 (0.61.1)

0.62 RMS Speed of an Ideal Gas


D

r
3RT
vrms = (0.62.1)
M

0.63 Translational Kinetic Energy


3
K̄ = kT (0.63.1)
2

0.64 Internal Energy of a Monatomic gas


3
Eint = nRT (0.64.1)
2

David S. Latchman ©2009


Molar Specific Heat at Constant Volume xlv
0.65 Molar Specific Heat at Constant Volume

Let us define, CV such that


Q = nCV ∆T (0.65.1)

Substituting into the First Law of Thermodynamics, we have

∆Eint + W = nCV ∆T (0.65.2)

At constant volume, W = 0, and we get

1 ∆Eint

FT
CV = (0.65.3)
n ∆T

Substituting eq. (0.64.1), we get

3
CV = R = 12.5 J/mol.K (0.65.4)
2
RA
0.66 Molar Specific Heat at Constant Pressure

Starting with
Q = nCp ∆T (0.66.1)

and
D

∆Eint = Q − W
⇒ nCV ∆T = nCp ∆T + nR∆T
∴ CV = Cp − R (0.66.2)

0.67 Equipartition of Energy

!
f
CV = R = 4.16 f J/mol.K (0.67.1)
2

where f is the number of degrees of freedom.

©2009 David S. Latchman


Thermodynamics & Statistical Mechanics

©2009
FT Degrees of Freedom Predicted Molar Specific Heats
CP = CV + R
RA
Molecule Translational Rotational Vibrational Total ( f ) CV
3 5
Monatomic 3 0 0 3 2
R 2
R
5 7
Diatomic 3 2 2 5 2
R 2
R
Polyatomic (Linear) 3 3 3n − 5 6 3R 4R
Polyatomic (Non-Linear) 3 3 3n − 6 6 3R 4R
Table 0.67.1: Table of Molar Specific Heats
D

David S. Latchman
xlvi
Adiabatic Expansion of an Ideal Gas xlvii
0.68 Adiabatic Expansion of an Ideal Gas
PV γ = a constant (0.68.1)
where γ = CCVP .
We can also write
TV γ−1 = a constant (0.68.2)

0.69 Second Law of Thermodynamics


Something.

FT
RA
D

©2009 David S. Latchman


xlviii Thermodynamics & Statistical Mechanics

FT
RA
D

David S. Latchman ©2009


Quantum Mechanics

0.70 Fundamental Concepts


1

0.71 Schrödinger Equation


Let us define Ψ to be
FT
Ψ = Ae−iω(t− v )
x

Simplifying in terms of Energy, E, and momentum, p, we get


(0.71.1)
RA
i(Et−px)
Ψ = Ae− ~ (0.71.2)
We obtain Schrödinger’s Equation from the Hamiltonian
H =T+V (0.71.3)
To determine E and p,
∂2 Ψ p2
= − Ψ (0.71.4)
∂x2 ~2
∂Ψ iE
= Ψ (0.71.5)
∂t ~
D

and
p2
H= +V (0.71.6)
2m
This becomes
EΨ = HΨ (0.71.7)
~ ∂Ψ ∂Ψ 2
EΨ = − p2 Ψ = −~2 2
i ∂t ∂x
The Time Dependent Schrödinger’s Equation is
∂Ψ ~ 2 ∂2 Ψ
i~ =− + V(x)Ψ (0.71.8)
∂t 2m ∂x2
The Time Independent Schrödinger’s Equation is
~ 2 ∂2 Ψ
EΨ = − + V(x)Ψ (0.71.9)
2m ∂x2
l Quantum Mechanics
0.71.1 Infinite Square Wells

Let us consider a particle trapped in an infinite potential well of size a, such that

for 0 < x < a


(
0
V(x) =
∞ for |x| > a,

so that a nonvanishing force acts only at ±a/2. An energy, E, is assigned to the system
such that the kinetic energy of the particle is E. Classically, any motion is forbidden
outside of the well because the infinite value of V exceeds any possible choice of E.
Recalling the Schrödinger Time Independent Equation, eq. (0.71.9), we substitute V(x)
and in the region (−a/2, a/2), we get

~2 d2 ψ
− = Eψ (0.71.10)

FT
2m dx2
This differential is of the form
d2 ψ
+ k2 ψ = 0 (0.71.11)
dx2
where r
2mE
k= (0.71.12)
~2
We recognize that possible solutions will be of the form
RA
cos kx and sin kx
As the particle is confined in the region 0 < x < a, we say

A cos kx + B sin kx for 0 < x < a


(
ψ(x) =
0 for |x| > a

We have known boundary conditions for our square well.


ψ(0) = ψ(a) = 0 (0.71.13)
D

It shows that
⇒ A cos 0 + B sin 0 = 0
∴A=0 (0.71.14)
We are now left with
B sin ka = 0
ka = 0; π; 2π; 3π; · · ·
(0.71.15)
While mathematically, n can be zero, that would mean there would be no wave function,
so we ignore this result and say

kn = for n = 1, 2, 3, · · ·
a

David S. Latchman ©2009


Schrödinger Equation li
Substituting this result into eq. (0.71.12) gives

nπ 2mEn
kn = = (0.71.16)
a ~
Solving for En gives
n2 π2 ~2
En = (0.71.17)
2ma2
We cna now solve for B by normalizing the function
Z a
a
|B|2 sin2 kxdx = |A|2 = 1
0 2
2
So |A|2 = (0.71.18)
a

FT
So we can write the wave function as
r
2 nπx
 
ψn (x) = sin (0.71.19)
a a

0.71.2 Harmonic Oscillators

Classically, the harmonic oscillator has a potential energy of


RA
1
V(x) = kx2 (0.71.20)
2
So the force experienced by this particle is
dV
F=− = −kx (0.71.21)
dx
where k is the spring constant. The equation of motion can be summed us as
d2 x
m = −kx (0.71.22)
dt2
D

And the solution of this equation is


 
x(t) = A cos ω0 t + φ (0.71.23)

where the angular frequency, ω0 is


r
k
ω0 = (0.71.24)
m
The Quantum Mechanical description on the harmonic oscillator is based on the eigen-
function solutions of the time-independent Schrödinger’s equation. By taking V(x)
from eq. (0.71.20) we substitute into eq. (0.71.9) to get
d2 ψ 2m k 2
!
mk 2 2E
 
= 2 x −E ψ= 2 x − ψ
dx2 ~ 2 ~ k

©2009 David S. Latchman


lii Quantum Mechanics
With some manipulation, we get
√
d2 ψ  mk 2 2E m 
r 
~
√ 2
=  x −  ψ
mk dx ~ ~ k 

This step allows us to to keep some of constants out of the way, thus giving us

mk 2
ξ2 = x (0.71.25)
~r
2E m 2E
and λ = = (0.71.26)
~ k ~ω0

This leads to the more compact

d2 ψ  2

FT

= ξ − λ ψ (0.71.27)
dξ2
where the eigenfunction ψ will be a function of ξ. λ assumes an eigenvalue anaglaous
to E.
From eq. (0.71.25), we see that the maximum value can be determined to be

mk 2
ξmax =
2
A (0.71.28)
RA
~
Using the classical connection between A and E, allows us to say

mk 2E
ξmax =
2
=λ (0.71.29)
~ k
From eq. (0.71.27), we see that in a quantum mechanical oscillator, there are non-
vanishing solutions in the forbidden regions, unlike in our classical case.
A solution to eq. (0.71.27) is
ψ(ξ) = e−ξ /2
2
(0.71.30)
D

where

= −ξe−ξ /2
2


dψ 2 −xi2 /2 −ξ2 /2
e−ξ /2
  2
and = ξ e − e = ξ 2
− 1
dξ2
This gives is a special solution for λ where

λ0 = 1 (0.71.31)

Thus eq. (0.71.26) gives the energy eigenvalue to be

~ω0 ~ω0
E0 = λ0 = (0.71.32)
2 2

David S. Latchman ©2009


Schrödinger Equation liii
−ξ2 /2
The eigenfunction e corresponds to a normalized stationary-state wave function
!1
mk 8 − √mk x2 /2~ −iE0 t/~
Ψ0 (x, t) = 2 2 e e (0.71.33)
π~
This solution of eq. (0.71.27) produces the smallest possibel result of λ and E. Hence,
Ψ0 and E0 represents the ground state of the oscillator. and the quantity ~ω0 /2 is the
zero-point energy of the system.

0.71.3 Finite Square Well

For the Finite Square Well, we have a potential region where


(
−V0 for −a ≤ x ≤ a
V(x) =

FT
0 for |x| > a
We have three regions

Region I: x < −a In this region, The potential, V = 0, so Schrödinger’s Equation be-


comes
~2 d2 ψ
− = Eψ
2m dx2
d2 ψ
RA
⇒ 2 = κ2 ψ
√ dx
−2mE
where κ=
~
This gives us solutions that are
ψ(x) = A exp(−κx) + B exp(κx)
As x → ∞, the exp(−κx) term goes to ∞; it blows up and is not a physically
realizable function. So we can drop it to get
D

ψ(x) = Beκx for x < −a (0.71.34)

Region II: −a < x < a In this region, our potential is V(x) = V0 . Substitutin this into
the Schrödinger’s Equation, eq. (0.71.9), gives
~2 d2 ψ
− − V0 ψ = Eψ
2m dx2
d2 ψ
or 2
= −l2 ψ
p dx
2m (E + V0 )
where l ≡ (0.71.35)
~
We notice that E > −V0 , making l real and positive. Thus our general solution
becomes
ψ(x) = C sin(lx) + D cos(lx) for −a < x < a (0.71.36)

©2009 David S. Latchman


liv Quantum Mechanics
Region III: x > a Again this Region is similar to Region III, where the potential, V = 0.
This leaves us with the general solution

ψ(x) = F exp(−κx) + G exp(κx)

As x → ∞, the second term goes to infinity and we get

ψ(x) = Fe−κx for x > a (0.71.37)

This gives us  κx

 Be for x < a
ψ(x) =  for 0 < x < a

D cos(lx) (0.71.38)

for x > a

 Fe−κx

FT
0.71.4 Hydrogenic Atoms

0.72 Spin
RA
3

0.73 Angular Momentum


4

0.74 Wave Funtion Symmetry


D

0.75 Elementary Perturbation Theory


6

David S. Latchman ©2009


Atomic Physics

0.76 Properties of Electrons


1

0.77 Bohr Model

FT
To understand the Bohr Model of the Hydrogen atom, we will take advantage of our
knowlegde of the wavelike properties of matter. As we are building on a classical
model of the atom with a modern concept of matter, our derivation is considered to be
RA
‘semi-classical’. In this model we have an electron of mass, me , and charge, −e, orbiting
a proton. The cetripetal force is equal to the Coulomb Force. Thus
1 e2 me v2
= (0.77.1)
4π0 r2 r
The Total Energy is the sum of the potential and kinetic energies, so
p2
E=K+U = − | f race2 4π0 r (0.77.2)
2me
We can further reduce this equation by subsituting the value of momentum, which we
D

find to be
p2 1 e2
= me v2 = (0.77.3)
2me 2 8π0 r
Substituting this into eq. (0.77.2), we get
e2 e2 e2
E= − =− (0.77.4)
8π0 r 4π0 r 8π0 r
At this point our classical description must end. An accelerated charged particle, like
one moving in circular motion, radiates energy. So our atome here will radiate energy
and our electron will spiral into the nucleus and disappear. To solve this conundrum,
Bohr made two assumptions.

1. The classical circular orbits are replaced by stationary states. These stationary
states take discreet values.
lvi Atomic Physics
2. The energy of these stationary states are determined by their angular momentum
which must take on quantized values of ~.
L = n~ (0.77.5)

We can find the angular momentum of a circular orbit.


L = m3 vr (0.77.6)
From eq. (0.77.1) we find v and by substitution, we find L.
r
m3 r
L=e (0.77.7)
4π0
Solving for r, gives
L2

FT
r= (0.77.8)
me e2 /4π0
We apply the condition from eq. (0.77.5)
n2 ~2
rn = = n2 a0 (0.77.9)
me e /4π0
2

where a0 is the Bohr radius.


a0 = 0.53 × 10−10 m (0.77.10)
RA
Having discreet values for the allowed radii means that we will also have discreet
values for energy. Replacing our value of rn into eq. (0.77.4), we get
!
me e2 13.6
En = − 2 = − 2 eV (0.77.11)
2n 4π0 ~ n

0.78 Energy Quantization


3
D

0.79 Atomic Structure


4

0.80 Atomic Spectra

0.80.1 Rydberg’s Equation


1 1 1
 
= RH 02 − 2 (0.80.1)
λ n n

David S. Latchman ©2009


Selection Rules lvii
where RH is the Rydberg constant.
For the Balmer Series, n0 = 2, which determines the optical wavelengths. For n0 = 3, we
get the infrared or Paschen series. The fundamental n0 = 1 series falls in the ultraviolet
region and is known as the Lyman series.

0.81 Selection Rules


6

0.82 Black Body Radiation

0.82.1

0.82.2
Plank Formula

FT
8π~ f3
u( f, T) = 3 h f /kT

Stefan-Boltzmann Formula
c e −1
(0.82.1)
RA
P(T) = σT4 (0.82.2)

0.82.3 Wein’s Displacement Law

λmax T = 2.9 × 10−3 m.K (0.82.3)

0.82.4 Classical and Quantum Aspects of the Plank Equation


D

Rayleigh’s Equation

8π f 2
u( f, T) = kT (0.82.4)
c3
We can get this equation from Plank’s Equation, eq. (0.82.1). This equation is a classical
one and does not contain Plank’s constant in it. For this case we will look at the
situation where h f < kT. In this case, we make the approximation

ex ' 1 + x (0.82.5)

Thus the demonimator in eq. (0.82.1) becomes

hf hf
eh f /kT − 1 ' 1 + −1= (0.82.6)
kT kT

©2009 David S. Latchman


lviii Atomic Physics
Thus eq. (0.82.1) takes the approximate form

8πh 3 kT 8π f 2
u( f, T) ' 3 f = 3 kT (0.82.7)
c hf c
As we can see this equation is devoid of Plank’s constant and thus independent of
quantum effects.

Quantum

At large frequencies, where h f > kT, quantum effects become apparent. We can
estimate that
eh f /kT − 1 ' eh f /kT (0.82.8)

FT
Thus eq. (0.82.1) becomes
8πh 3 −h f /kT
u( f, T) ' f e (0.82.9)
c3

0.83 X-Rays

0.83.1 Bragg Condition


RA
2d sin θ = mλ (0.83.1)
for constructive interference off parallel planes of a crystal with lattics spacing, d.

0.83.2 The Compton Effect

The Compton Effect deals with the scattering of monochromatic X-Rays by atomic
targets and the observation that the wavelength of the scattered X-ray is greater than
the incident radiation. The photon energy is given by
D

hc
E = hυ = (0.83.2)
λ
The photon has an associated momentum

E= pc (0.83.3)
E hυ h
⇒p = = = (0.83.4)
c c λ
The Relativistic Energy for the electron is

E2 = p2 c2 + m2e c4 (0.83.5)

where
p − p0 = P (0.83.6)

David S. Latchman ©2009


Atoms in Electric and Magnetic Fields lix
Squaring eq. (0.83.6) gives
p2 − 2p · p0 + p02 = P2 (0.83.7)
Recall that E = pc and E 0 = cp0 , we have

c2 p2 − 2c2 p · p0 + c2 p02 = c2 P2
E 2 − 2E E 0 cos θ + E 02 = E2 − m2e c4 (0.83.8)

Conservation of Energy leads to

E + me c2 = E 0 + E (0.83.9)

Solving

E − E 0 = E − me c2
E 2 − 2E E 0 + E 0 = E2 − 2Eme c2 + m2e c4 (0.83.10)

FT
2E E 0 − 2E E 0 cos θ = 2Eme c2 − 2m2e c4 (0.83.11)

Solving leads to
h
∆λ = λ0 − λ = (1 − cos θ) (0.83.12)
me c
where λc = h
me c
is the Compton Wavelength.

h
RA
λc = = 2.427 × 10−12 m (0.83.13)
me c

0.84 Atoms in Electric and Magnetic Fields

0.84.1 The Cyclotron Frequency

A test charge, q, with velocity v enters a uniform magnetic field, B. The force acting on
the charge will be perpendicular to v such that
D

FB = qv × B (0.84.1)

or more simply FB = qvB. As this traces a circular path, from Newton’s Second Law,
we see that
mv2
FB = = qvB (0.84.2)
R
Solving for R, we get
mv
R= (0.84.3)
qB
We also see that
qB
f = (0.84.4)
2πm
The frequency is depends on the charge, q, the magnetic field strength, B and the mass
of the charged particle, m.

©2009 David S. Latchman


lx Atomic Physics
0.84.2 Zeeman Effect

The Zeeman effect was the splitting of spectral lines in a static magnetic field. This is
similar to the Stark Effect which was the splitting in the presence in a magnetic field.
In the Zeeman experiment, a sodium flame was placed in a magnetic field and its
spectrum observed. In the presence of the field, a spectral line of frequency, υ0 was
split into three components, υ0 − δυ, υ0 and υ0 + δυ. A classical analysis of this effect
allows for the identification of the basic parameters of the interacting system.
The application of a constant magnetic field, B, allows for a direction in space in which
the electron motion can be referred. The motion of an electron can be attributed to a
simple harmonic motion under a binding force −kr, where the frequency is
r
1 k
υ0 = (0.84.5)

FT
2π me
The magnetic field subjects the electron to an additional Lorentz Force, −ev × B. This
produces two different values for the angular velocity.
v = 2πrυ
The cetripetal force becomes
me v2
= 4π2 υ2 rme
r
RA
Thus the certipetal force is
4π2 υ2 rme = 2πυreB + kr for clockwise motion
4π2 υ2 rme = −2πυreB + kr for counterclockwise motion
We use eq. (0.84.5), to emiminate k, to get
eB
υ2 − υ − υ0 = 0 (Clockwise)
2πme
eB
υ2 + υ − υ0 = 0 (Counterclockwise)
2πme
D

As we have assumed a small Lorentz force, we can say that the linear terms in υ are
small comapred to υ0 . Solving the above quadratic equations leads to
eB
υ = υ0 + for clockwise motion (0.84.6)
4πme
eB
υ = υ0 − for counterclockwise motion (0.84.7)
4πme
We note that the frequency shift is of the form
eB
δυ = (0.84.8)
4πme
If we view the source along the direction of B, we will observe the light to have two
polarizations, a closckwise circular polarization of υ0 + δυ and a counterclosckwise
circular polarization of υ0 − δυ.

David S. Latchman ©2009


Atoms in Electric and Magnetic Fields lxi
0.84.3 Franck-Hertz Experiment

The Franck-Hertz experiment, performed in 1914 by J. Franck and G. L. Hertz, mea-


sured the colisional excitation of atoms. Their experiement studied the current of
electrons in a tub of mercury vapour which revealed an abrupt change in the current
at certain critical values of the applied voltage.2 They interpreted this observation as
evidence of a threshold for inelastic scattering in the colissions of electrons in mer-
cury atoms.The bahavior of the current was an indication that electrons could lose
a discreet amount of energy and excite mercury atoms in their passage through the
mercury vapour. These observations constituted a direct and decisive confirmation of
the existence os quantized energy levels in atoms.

FT
RA
D

2
Put drawing of Franck-Hertz Setup

©2009 David S. Latchman


lxii Atomic Physics

FT
RA
D

David S. Latchman ©2009


Special Relativity

0.85 Introductory Concepts

0.85.1 Postulates of Special Relativity

We can define
γ= q
FT
1. The laws of Physics are the same in all inertial frames.

2. The speed of light is the same in all inertial frames.

1
(0.85.1)
RA
u2
1− c2

0.86 Time Dilation

∆t = γ∆t0 (0.86.1)
where ∆t0 is the time measured at rest relative to the observer, ∆t is the time measured
in motion relative to the observer.
D

0.87 Length Contraction

L0
L= (0.87.1)
γ
where L0 is the length of an object observed at rest relative to the observer and L is the
length of the object moving at a speed u relative to the observer.

0.88 Simultaneity

4
lxiv Special Relativity
0.89 Energy and Momentum

0.89.1 Relativistic Momentum & Energy

In relativistic mechanics, to be conserved, momentum and energy are defined as

Relativistic Momentum
p̄ = γmv̄ (0.89.1)

Relativistic Energy
E = γmc2 (0.89.2)

FT
0.89.2 Lorentz Transformations (Momentum & Energy)

E
 
p0x= γ px − β (0.89.3)
c
py = py
0
(0.89.4)
= pz
p0z (0.89.5)
0
RA
E E
 
=γ − βpx (0.89.6)
c c

0.89.3 Relativistic Kinetic Energy

K = E − mc2 (0.89.7)
 
 
 1 
= mc  q
2
− 1 (0.89.8)
D

v2
1−
 
c2

= mc2 γ − 1

(0.89.9)

0.89.4 Relativistic Dynamics (Collisions)

∆E
 
∆P0x = γ ∆Px − β (0.89.10)
c
∆P y = ∆P y
0
(0.89.11)
∆P0z = ∆Pz (0.89.12)
∆E0 ∆E
 
=γ − β∆Px (0.89.13)
c c

David S. Latchman ©2009


Four-Vectors and Lorentz Transformation lxv
0.90 Four-Vectors and Lorentz Transformation
We can represent an event in S with the column matrix, s,

 x 
 
 y 
s =  (0.90.1)
 
 z 

 
ict

A different Lorents frame, S0 , corresponds to another set of space time axes so that
 0 
 x 
 y0 
s0 =  0  (0.90.2)
 z 
 0 

FT
ict

The Lorentz Transformation is related by the matrix

 x   γ
 0  
0 0 iγβ   x 
 
 y0   0 1 0 0   y 
 0  =  (0.90.3)
     
 z   0 0 1 0   z 
  
ict0 −iγβ 0 0 γ ict
RA
We can express the equation in the form

s0 = L s (0.90.4)

The matrix L contains all the information needed to relate position four–vectors for
any given event as observed in the two Lorentz frames S and S0 . If we evaluate

 x 
 
h i  y 
sT s = x y z ict   = x2 + y2 + z2 − c2 t2 (0.90.5)
 z 
 
ict
D

Similarly we can show that

s0T s0 = x02 + y02 + z02 − c2 t02 (0.90.6)

We can take any collection of four physical quantities to be four vector provided that
they transform to another Lorentz frame. Thus we have

 bx 
 
 b 
b =  y  (0.90.7)
 
 bz 
 
ibt

this can be transformed into a set of quantities of b0 in another frame S0 such that it
satisfies the transformation
b0 = L b (0.90.8)

©2009 David S. Latchman


lxvi Special Relativity
Looking at the momentum-Energy four vector, we have

 px 
 
 p 
p =  y  (0.90.9)
 
 pz 
 
iE/c
Applying the same transformation rule, we have
p0 = L p (0.90.10)
We can also get a Lorentz-invariation relation between momentum and energy such
that
p0T p0 = pT p (0.90.11)
The resulting equality gives

FT
E02 E2
x + p y + pz −
p02 = + +
02 02 2 2 2
px p y p z − (0.90.12)
c2 c2

0.91 Velocity Addition


v−u
v0 = (0.91.1)
1 − uv
RA
c2

0.92 Relativistic Doppler Formula


r r
c+u c−u
ῡ = υ0 let r = (0.92.1)
c−u c+u
We have
ῡreceding = rυ0 red-shift (Source Receding) (0.92.2)
D

υ0
ῡapproaching = blue-shift (Source Approaching) (0.92.3)
r

0.93 Lorentz Transformations


Given two reference frames S(x, y, z, t) and S0 (x0 , y0 , z0 , t0 ), where the S0 -frame is moving
in the x-direction, we have,
x0 = γ (x − ut) x = (x0 − ut0 ) (0.93.1)
y0 = y y = y0 (0.93.2)
z0 = y y0 = y (0.93.3)
u u 0
   
t = γ t − 2x
0
t = γ t + 2x
0
(0.93.4)
c c

David S. Latchman ©2009


Space-Time Interval lxvii
0.94 Space-Time Interval

(∆S)2 = (∆x)2 + ∆y 2 + (∆z)2 − c2 (∆t)2



(0.94.1)
Space-Time Intervals may be categorized into three types depending on their separa-
tion. They are

Time-like Interval

c2 ∆t2 > ∆r2 (0.94.2)


∆S > 0
2
(0.94.3)

When two events are separated by a time-like interval, there is a cause-effect


relationship between the two events.

FT
Light-like Interval

c2 ∆t2 = ∆r2 (0.94.4)


S2 = 0 (0.94.5)

Space-like Intervals
RA
c2 ∆t2 < ∆r2 (0.94.6)
∆S < 0 (0.94.7)
D

©2009 David S. Latchman


lxviii Special Relativity

FT
RA
D

David S. Latchman ©2009


Laboratory Methods

0.95 Data and Error Analysis

0.95.1 Addition and Subtraction

The Error in x is

FT
x=a+b−c

(δx)2 = (δa)2 + (δb)2 + (δc)2


(0.95.1)

(0.95.2)
RA
0.95.2 Multiplication and Division

a×b
x= (0.95.3)
c
The error in x is
!2
δx δa δb δc
 2  2  2
= + + (0.95.4)
x a b c
D

0.95.3 Exponent - (No Error in b)

x = ab (0.95.5)
The Error in x is
δx δa
 
=b (0.95.6)
x a

0.95.4 Logarithms

Base e

x = ln a (0.95.7)
lxx Laboratory Methods
We find the error in x by taking the derivative on both sides, so

d ln a
δx = · δa
da
1
= · δa
a
δa
= (0.95.8)
a

Base 10

x = log10 a (0.95.9)

FT
The Error in x can be derived as such

d(log a)
δx = δa
da
ln a
ln 10
= δa
da
1 δa
=
ln 10 a
δa
RA
= 0.434 (0.95.10)
a

0.95.5 Antilogs

Base e

x = ea (0.95.11)
We take the natural log on both sides.
D

ln x = a ln e = a (0.95.12)

Applaying the same general method, we see

d ln x
δx = δa
dx
δx
⇒ = δa (0.95.13)
x

Base 10

x = 10a (0.95.14)

David S. Latchman ©2009


Instrumentation lxxi
We follow the same general procedure as above to get
log x = a log 10
log x
δx = δa
dx
1 d ln a
δx = δa
ln 10 dx
δx
= ln 10δa (0.95.15)
x

0.96 Instrumentation
2

0.97
3

0.98
Radiation Detection

Counting Statistics
FT
RA
Let’s assume that for a particular experiment, we are making countung measurements
for a radioactive source. In this experiment, we recored N counts in time T. The
counting rate for this trial is R = N/T. This rate should be close to the average
√ rate, R̄.
The standard deviation or the uncertainty of our count is a simply called the N rule.
So √
σ= N (0.98.1)
Thus we can report our results as

D

Number of counts = N ± N (0.98.2)


We can find the count rate by dividing by T, so

N N
R= ± (0.98.3)
T T
δN
The fractional uncertainty of our count is N
. We can relate this in terms of the count
rate.
δN
δR T δN
= N
=
R T
N

N
=
N
1
= (0.98.4)
N

©2009 David S. Latchman


lxxii Laboratory Methods
We see that our uncertainty decreases as we take more counts, as to be expected.

0.99 Interaction of Charged Particles with Matter


5

0.100 Lasers and Optical Interferometers


6

FT
0.101 Dimensional Analysis
Dimensional Analysis is used to understand physical situations involving a mis of
different types of physical quantities. The dimensions of a physical quantity are
associated with combinations of mass, length, time, electric charge, and temperature,
represented by symbols M, L, T, Q, and θ, respectively, each raised to rational powers.
RA
0.102 Fundamental Applications of Probability and Statis-
tics
8
D

David S. Latchman ©2009


GR9677 Exam Solutions

0.103 Discharge of a Capacitor


The voltage of a capacitor follows an exponential decay

FT
t
 
V(t) = V0 exp − (0.103.1)
RC
When the switch is toggled in the a position, the capacitor is quickly charged and the
potential across its plates is V. r is small and we assume that the potential difference
across it is negligible. When the switch is toggled on the b position, the voltage across
the capacitor begins to decay. We can find the current through the resistor, R, from
Ohm’s Law
RA
V(t) t
 
I(t) = = V0 exp − (0.103.2)
R RC
At t = 0 V0 = V. Graph B, shows an exponential decay.
Answer: (B)

0.104 Magnetic Fields & Induced EMFs


D

We have a circuit loop that is placed in a decaying magnetic field where the field
direction acts into the page. We have two currents in the circuit. The first is due to the
battery and the other is an induced current from the changing magnetic field. We can
easily determine the current of the cell from Ohm’s Law.

V 5.0
Ic = = A (0.104.1)
R 10
The induced EMF from the magnetic field is found from Faraday’s Law of Induction


E =− (0.104.2)
dt
where Φ is the magnetic flux.
dΦ dB
=A (0.104.3)
dt dt
lxxiv GR9677 Exam Solutions
The area of our loop, A = 10 × 10cm . So the induced EMF is
2

E = −100 × 10−4 × 150 = 1.5 V (0.104.4)

The field acts into the page, we consider this a negative direction, it’s decaying, also
negative. So
negative × negative = positive (0.104.5)
Faraday’s Law of Induction has a negative sign. So we expect our EMF to be negative.
Using the Right Hand Grip Rule, and pointing our thumb into the page, our fingers
curl in the clockwise direction. So we see that the current from our cell goes in the
counter-clockwise direction and the induced current in the clockwise direction; they
oppose each other. The total EMF is

V = 5.0 − 1.5 = 3.5 volt (0.104.6)

The current through the resistor is

Answer: (B)
I=
10

FT
3.5
= 0.35 A (0.104.7)
RA
0.105 A Charged Ring I

The Electric Potential is


Q
V= (0.105.1)
4π0 r
The distance, r, of P from the charged ring is found from the pythagorean theorem

r2 = R2 + x2 (0.105.2)
D

Plugging this into the above equation yields

Q
V= √ (0.105.3)
4π0 R2 + x2

Answer: (B)

0.106 A Charged Ring II

The force a small charge, q experiences if placed in the center of the ring can be found
from Coulomb’s Law
qQ
F= (0.106.1)
4π0 R2

David S. Latchman ©2009


Forces on a Car’s Tires lxxv
If it undergoes small oscillations, R >> x, then

F = mRω2 (0.106.2)

Equating the two equations, and solving for ω, we get


r
qQ
ω= (0.106.3)
4π0 mR3
Answer: (A)

0.107 Forces on a Car’s Tires

FT
The horizontal force on the car’s tires is the sum of two forces, the cetripetal force and
the frictional force of the road. The cetripetal force acts towards the center, FA , while
the frictional force acts in the forwards direction, FC . If it’s not immediately clear why
it acts in the forward direction, the tires, as they rotate, exert a backward force on the
road. The road exerts an equal and opposite force on the tires, which is in the forward
direction3 So the force on the tires is the sum of these forces, FA and FC , which is FB
Answer: (B)
RA
0.108 Block sliding down a rough inclined plane
We are told several things in this question. The first is that the block attains a constant
speed, so it gains no kinetic energy; all its potential energy is lost due to friction.
Answer: (B)

0.108.1 Calculation
D

If you’d like a more rigorous proof, not something you might do in the exam. The
work done by the frictional force, Fr is
Z
W = Fr dx (0.108.1)

Fr acts along the direction of the incline and is equal to

Fr = mg sin θ (0.108.2)

The distance the force acts is


h
x= (0.108.3)
sin θ
3
Sometimes the frictional force can act in the direction of motion. This is one such case.

©2009 David S. Latchman


lxxvi GR9677 Exam Solutions
So the work done is

W = Fr · x
h
= mg sin θ ×
sin θ
= mgh

Answer: (B)

0.109 Collision of Suspended Blocks


We are told that the ball collides elastically with the block, so both momentum and

FT
energy are conserved. As the ball falls from a height, h, its potential energy is converted
to kinetic energy
1
mgh = mv21
2
v1 = 2gh
2
(0.109.1)

Momentum is conserved, so

mv1 = mv2 + 2mv3


RA
v1 = v2 + 2v3 (0.109.2)

Energy is also conserved, so


1 2 1 2 1
mv = mv + 2mv23
2 1 2 2 2
v21 = v22 + 2v23 (0.109.3)

Squaring eq. (0.109.2) and equating with eq. (0.109.3) gives

v21 = (v2 + 2v3 )2


D

∴ 2v2 = −v3 (0.109.4)

Substituting this into eq. (0.109.2) gives

v1 = v2 + 2v3
= 3v2
⇒ v21 = 9v22 (0.109.5)

The 2m block’s kinetic energy is converted to potential energy as it rises to a height of


h2 . Thus
1
mgh2 = mv22
2
∴ v22 = 2gh2 (0.109.6)

David S. Latchman ©2009


Damped Harmonic Motion lxxvii
We see that
2gh
= 2gh2
9
h
⇒ = h2 (0.109.7)
9
Answer: (A)

0.110 Damped Harmonic Motion


From section 0.4.4, we see that the frequency of a damped oscillator is
s
!2
b

FT
ω0 = ω20 − (0.110.1)
2m
This shows that the damped frequency will be lower than the natural frequency, ω0 , or
its period, T0 , will be longer.
Answer: (A)
RA
0.111 Spectrum of the Hydrogen Atom
The hydrogen spectrum can be found by the emperical Rydberg equation
 
1  1 1 
= RH  2 − 2  (0.111.1)
λ n f ni

where ni and n f are the intial and final states respectively. The longest wavelength, or
the smallest energy transition, would represent the transition n1 = n f + 1.
For the Lyman series, n f = 1, which lies in the ultra-violet spectrum, we have
D

1 1 1 3
 
= RH 2 − 2 = RH (0.111.2)
λL 1 2 4
For the Balmer series, n f = 2, which lies in the optical spectrum, we have
1 1 1 5
 
= RH 2 − 2 = RH (0.111.3)
λB 2 3 36
Dividing eq. (0.111.3) by eq. (0.111.2), we get
5
λL R
36 H 5
= = (0.111.4)
λB 3
R
4 H
27

Answer: (A)4
4
The other transition, the Paschen series, n f = 3, lies in the infra-red region of the spectrum.

©2009 David S. Latchman


lxxviii GR9677 Exam Solutions
0.112 Internal Conversion
We are lucky that they actually tell us what the internal conversion process is. From
this we gather that the inner most electron has left its orbit and the most likely outcome
will be for the remaining electrons to ‘fall’ in and take its place. These transitions will
result in the emission of X-ray photons5 .
Answer: (B)

0.113 The Stern-Gerlach Experiment


The description of the experiment in the question is the Stern-Gerlach Experiment.

FT
In this experiment, we expect the electrons to be deflected vertically into two beams
representing spin-up and spin-down electrons.
Answer: (D)

0.114 Positronium Ground State Energy


RA
The positronium atom consists of an electron and a positron in a bound state. Classi-
cally, this atom looks like two planets orbiting a central point or center of mass. We
need to reduce this system to an equivalent one where an electron cirlces a central
mass. We call this equivalent system the reduced mass of the two body system. This is
me M
µ= (0.114.1)
M + me
The energy levels in terms of the reduced mass is defined as

Z2 µ
En = − 2 E0 (0.114.2)
D

n me

The reduced mass of positronium is


µ me 1
= = (0.114.3)
me 2me 2
and the ground state of this atom, Z = 1 and n = 1. The ground state energy of
Hydrogen is 13.6eV.
eq. (0.114.2) becomes
1
E1 = − 13.6eV = −6.8 eV (0.114.4)
2
Answer: (C)
5
This can also result in the emission of an Auger Electron

David S. Latchman ©2009


Specific Heat Capacity and Heat Lost lxxix
0.115 Specific Heat Capacity and Heat Lost

In this question, you are being asked to put several things together. Here, we are told,
a heater is placed into the water but the water does not boil or change temperture. We
can assume that all of the supplied heat by the heater is lost and we infer from the
power of the heater that 100 Joules is lost per second.
The energy to change water by one degree is derived from its specific heat capacity.

E = SHC × Mass × Temp. Diff. = 4200 × 1 × 1 = 4200 J (0.115.1)

So the time to loose 4200 Joules of heat is

E 4200
t= = = 42 s (0.115.2)

FT
P 100

Answer: (B)

0.116 Conservation of Heat


RA
Assuming that little to no heat is lost to the environment, the two blocks will exchange
heat until they are both in thermal equilibrium with each other. As they have the same
masses we expect the final temperature to be 50°C. We can, of course, show this more
rigorously where both blocks reach a final temperature, T f . The intial temperatures of
blocks I and II are T1 = 100°Cand T2 = 0°C, respectively.

Heat Lost by Block I = Heat gained by Block II


   
0.1 × 103 × 1 × 100 − T f = 0.1 × 103 × 1 × T f − 0
 
2 0.1 × 103 T f = 10 × 103
D

∴ T f = 50 °C

Thus the heat exchanged is

0.1 × 103 × 1 × (100 − 50) = 5 kcal (0.116.1)

Answer: (D)

0.117 Thermal Cycles

We are told the cycle is reversible and moves from ABCA. We can examine each path
and add them to get the total work done.

©2009 David S. Latchman


lxxx GR9677 Exam Solutions
Path A → B is an isothermal process
Z V2
nRT
WA→B = P · dV where P =
V1 V
Z V2
dV
= nRTh
V1 V
V2
 
= nRTh ln (0.117.1)
V1
Path B → C is an isobaric process
Z V1
WB→C = P2 dV where P2 V2 = nRTh
V2

FT
= P2 (V1 − V2 ) and P2 V1 = nRTc
= nR (Tc − Th ) (0.117.2)

and Path C → A
Z
WC→A = P1 dV where dV = 0

=0 (0.117.3)
RA
Adding the above, we get

W = WA→B + WB→C + WC→A


V2
 
= nRTh ln + nR (Tc − Th ) (0.117.4)
V1
where n = 1 mole.
V2
 
W = RTh ln − R (Th − Tc ) (0.117.5)
V1
Answer: (E)
D

0.118 Mean Free Path


The mean free path of a particle, be it an atom, molecule or photon, is the average
distance travelled between collisions. We are given the equation as

1
`= (0.118.1)
ησ

where η is the number desnity and σ is the collision cross section. The number density
works out to be
N
η= (0.118.2)
V

David S. Latchman ©2009


Probability lxxxi
where N is the number of molecules and V is the volume. We can determine this from
the ideal gas law,

PV = NkT
N P
∴η= = (0.118.3)
V kT

The collision cross section is the area through which a particle can not pass without
colliding. This works out to be
σ = πd2 (0.118.4)

Now we can write eq. (0.118.1) in terms of variables we know

kT
`= (0.118.5)

FT
πPd2

As air is composed mostly of Nitrogen, we would have used the diameter of Nitrogen
in our calculations. This is approximately d = 3.1 Å. Plugging in the constants given
we have

(1.38 × 1023 )(300)


`=
π × 1.0 × 105 (3.1 × 1010 )2
RA
= 1.37 × 10−7 m

As we don’t have a calculator in the exam, we can estimate by adding the indices in
our equation,
− 23 + 2 − 5 + 20 = −6 (0.118.6)

So we expect our result to be in the order of 1 × 10−6 m. We choose (B).


Answer: (B)
D

0.119 Probability

The probability of finding a particle in a finite interval between two points, x1 and x2 ,
is
Z 4
P(2 ≤ x ≤ 4) = |Ψ(x)|2 dx (0.119.1)
2

with the normalization condition,


Z +∞
|Ψ(x)|2 dx = 1 (0.119.2)
−∞

We can tally the values given to us on the graph

©2009 David S. Latchman


lxxxii GR9677 Exam Solutions
x Ψ Ψ2
1 1 1
2 1 1
3 2 4
4 3 9
5 1 1
6 0 0
Total 16

Table 0.119.1: Table of wavefunction amplitudes

FT
The probability of finding the particle between (2 ≤ x ≤ 4) is

22 + 32
P(2 ≤ x ≤ 4) =
12 + 12 + 22 + 32 + 12 + 02
4+9
=
1+1+4+9+1+0
13
= (0.119.3)
16
RA
Answer: (E)

0.120 Barrier Tunneling

Classically, if a particle didn’t have enough kinetic energy, it would just bounce off
the wall but in the realm of Quantum Mechanics, there is a finite probability that the
particle will tunnel through the barrier and emerge on the other side. We expect to see
a few things. The wave function’s amplitide will be decreased from x > b and to decay
D

exponentially from a < x < b. We see that choice (C) has these characteristics.
Answer: (C)

0.121 Distance of Closest Appraoch

This question throws a lot of words at you. The α-particle with kietic energy 5 MeV is
shot towards an atom. If it goes towards the atom it will slow down, loosing kinetic
energy and gaining electrical potential energy. The α-particle will then be repelled by
the Ag atom. Thus
1 Q1 Q2
U= = KE (0.121.1)
4π0 D

David S. Latchman ©2009


Collisions and the He atom lxxxiii
Where D is the distance of closest approach, q1 = ze and q2 = Ze. We are given z = 2
for the alpha particle and Z = 50 for the metal atom. Plugging in all of this gives us
1 (2e)(50e)
D=
4π0 5 × 106 e
1 100e
=
4π0 5 × 106
1 100 × 1.6 × 10−19
=
4π × 8.85 × 10−12 5 × 106
≈ 0.3 × 10−13 m (0.121.2)

Answer: (B)

FT
0.122 Collisions and the He atom
As the collision is elastic, we know that both momentum and kinetic energy is con-
served. So conservation of momentum shows
4uv = (−0.6)(4u)v + MV
⇒ 6.4 uv = MV (0.122.1)
RA
Conservation of Energy shows that
1 1 1
(4 u)v2 = (4 u)(0.6v)2 + MV 2
2h i 2 2
4 u 0.64v = MV
2 2
(0.122.2)

Solving for M
6.42 u
M= = 16 u (0.122.3)
4(0.64)
We see that this corresponds to an Oxygen atom, mass 16 u.
D

Answer: (D)

0.123 Oscillating Hoops


We are given the period of our physical pendulum, where
s
I
T = 2π (0.123.1)
mgd

where I is the moment of inertia and d is the distance of the pivot from the center of
mass. The moment of inertia of our hoop is
Icm = Mr2 (0.123.2)

©2009 David S. Latchman


lxxxiv GR9677 Exam Solutions
The moment of inertia of the hoop hanging by a nail is found from the Parallex Axis
Theorem
I = Icm + Md2 = Mr2 + Mr2 = 2Mr2 (0.123.3)
Plugging this into the first equation gives
s
2Mr2
T = 2π
Mgr
s
2r
= 2π
g
r
2 × 20 × 10−2
≈ 2π
10
= 4π × 10 ≈ 1.2 s
−1

FT
Answer: (C)

0.124 Mars Surface Orbit


If a body travels forward quickly enough that it follows the planet’s curvature it is in
orbit. We are told that in the case of Mars, there is a 2.0 meter drop for every 3600 meter
RA
horizontal distance. We are also told that the acceleration due to gravity on Mars is
gM = 0.4g. So the time to drop a distance of 2.0 meters is
1
s= gM t2
2
⇒ t = 1s (0.124.1)
So the horizontal speed is
3600
vx = m/s (0.124.2)
1
Answer: (C)
D

0.125 The Inverse Square Law


Choice A Energy will be conserved. This isn’t dependent on an inverse square law.
Choice B Momentum is conserved. This also isn’t dependent on the inverse square
law.
Choice C This follows from Kepler’s Law
 2
2π Gm1 m2
mr =
T r2+
⇒ T ∝ r(3+)/2 (0.125.1)

David S. Latchman ©2009


Charge Distribution lxxxv
6
Choice D This is FALSE. This follows from Bertrand’s Theorem , which states that
only two types of potentials produce stable closed orbits

1. An inverse square central force such as the gravitational or electrostatic


potential.
−k
V(r) = (0.125.2)
r
2. The radial Harmonic Oscillator Potential
1
V(r) = kr2 (0.125.3)
2
Choice E A stationary circular orbit occurs under special conditions when the central
force is equal to the centripetal force. This is not dependent on an inverse square
law but its speed.

Answer: (D)

0.126 Charge Distribution


FT
An inportant thing to keep in mind is that charge will distribute itself evenly throughout
RA
the conducting spheres and that charge is conserved.

Step I: Uncharged Sphere C touches Sphere A

A B C
Q Q
Q
2 2

Step II: Sphere C is touched to Sphere B

A B C
D

Q 3Q 3Q
2 4 4

The initial force between A and B is


kQ2
F= (0.126.1)
r2
The final force between A and B is
k Q2 3Q
4 3
Ff = = F (0.126.2)
r2 8
Answer: (D)
6
Add reference here

©2009 David S. Latchman


lxxxvi GR9677 Exam Solutions
0.127 Capacitors in Parallel
We have one capacitor, C1 connected to a battery. This capacitor gets charged and
stores a charge, Q0 and energy, U0 .
Q0 = C1 V (0.127.1)
1
U0 = C1 V 2 (0.127.2)
2
When the switch is toggled in the on position, the battery charges the second capacitor,
C2 . As the capacitors are in parallel, the potential across them is the same. As C1 = C2 ,
we see that the charges and the energy stored across each capacitor is the same. Thus
Q1 = C1 V1 Q2 = C2 V2
Q2 = Q1

FT
1 1
U1 = C1 V12 U2 = C2 V22
2 2
U2 = U1
We see that
U1 + U2 = C1 V12 = 2U0 (0.127.3)
We can also analyze this another way. The two capacitors are in parallel, so their net
capacitance is
RA
CT = C1 + C2 = 2C1 (0.127.4)
So the total charge and energy stored by this parallel arrangemt is
Q = CT V1 = 2C1 V1
1
UT = 2C1 V12 = 2U0
2
Of all the choices, only (E) is incorrect.
Answer: (E)
D

0.128 Resonant frequency of a RLC Circuit


The circuit will be best ‘tuned’ when it is at its resonant frequence. This occurs when
the impedances for the capacitor and inductor are equal. Thus
1
XC = and XL = ωL (0.128.1)
ωC
When they are equal
XC = XL
1
= XL = ωL
ωC
1
∴ω= √ (0.128.2)
LC

David S. Latchman ©2009


Graphs and Data Analysis lxxxvii
Solving for C,
ω2 L
C=
1
= π2 × (103.7 × 106 )2 × 2.0 × 10−6
4
≈ 0.125 × 10−11 F (0.128.3)
Answer: (C)

0.129 Graphs and Data Analysis


It is best to analyse data is they are plotted on straight line graphs of the form, y = mx+c.

FT
This way we can best tell how well our data fits, etc.7

A We want a plot of activity, dN


dt
vs. time, t. If we were to plot this as is, we would
get an exponential curve. To get the straight line graph best suited for further
analysis, we take the logs on both sides.
dN
∝ e−2t
" dt#
dN
log = log e−2t
RA
dt
" #
dN
log = −2t (0.129.1)
dt
h i
We have a Semilog graph with a plot of log dN
dt
on the y-axis, t on the x-axis with
a gradient of 2.
B This is already a linear equation we can plot with the data we already have. No
need to manipulate it in any way.
C We take logs on bot sides of the equation to get
D

s ∝ t2
log s = 2 log t (0.129.2)
We can plot log s vs. log t. This gives a linear equation with log s on the y-axis
and log t on the x-axis and a gradient of 2.
D Again, we take logs on both sides of the equation
Vout 1

Vin ω
" #
V
log out = − log ω
Vin
7
This is of course with nothing but a sheet of graph paper and calculator and without the help of
computers and data analysis software.

©2009 David S. Latchman


lxxxviii V  GR9677 Exam Solutions
We have a log-log plot of log out on the y-axis and log ω on the x-axis with a
V
in
gradient of -1. We see that this choice is INCORRECT.
E As with the other choices, we take logs on both sides and get
P ∝ T4
log P = 4 log T
This can be plotted on a log-log graph with log P on the y-axis and log T on the
x-axis and a gradient of 4.

Answer: (D)

0.130 Superposition of Waves

The frequency is
T=
0.5 cm ms−1 FT
As the question states, we can see the superposition of the two waves. For the higher
frequency wave, we see that the period on the oscilloscope is about 1cm. This works
out to be a period of
1 cm
= 2.0 ms (0.130.1)
RA
1
f =
2.0 × 10−3
= 500 Hz (0.130.2)
We can measure the amplitude of this oscillation by measuring the distance from crest
to trough. This is approximately (2 − 1)/2, thus8
A = 1 cm × 2.0 V cm−1
≈ 2.0 V (0.130.3)
For the longer period wave, we notice that approximately a half-wavelength is dis-
played, is 2(4.5 − 1.5) = 6 cm. The period becomes
D

6.0 cm
T=
0.5 cm/ms
= 12.0 ms (0.130.4)
Thus the frequency is
1
f =
T
1
= = 83 Hz (0.130.5)
12.0 × 10−3
We see that (D) matches our calculations.
Answer: (D)
8
If you happened to have worked this one first you’ll notice that only choice (D) is valid. You can
stop and go on to the next question.

David S. Latchman ©2009


The Plank Length lxxxix
0.131 The Plank Length

This question is best analysed through dimensional analysis; unless of course you’re
fortunate to know the formula for the Plank Length. We are told that

G = 6.67 × 10−11 m3 kg−1 s−2


6.63 × 10−34 −1
~= Js

c = 3.0 × 108 m s−1

We can substitute the symbols for Length, L, Mass, M and Time, T. So the dimensions
of our constants become

FT
G = L3 M−1 T−2
~ = ML2 T−1
c = LT−1
`p = L

Our Plank Length is in the form


`p = Gx ~ y cz
RA
Dimensional Analysis Gives
 x  y  z
L = L3 M−1 T−2 ML2 T−1 LT−1

We get

L
3x + 2y + z = 1
D

M
−x+y=0

T
z = −3x

Solving, we get
1 1 3
x= y= z=−
2 2 2
Thus r
G~
`p =
c3
Answer: (E)

©2009 David S. Latchman


xc GR9677 Exam Solutions
0.132 The Open Ended U-tube
We recall that the pressure throughout a fluid is equal throughout the fluid. As the
system is in equlibrium, the pressure on the left arm is equal to the pressure on the
right arm. We can set up an equation such that

ρ2 g5 + ρ1 g (h1 − 5) = ρ1 gh2 (0.132.1)

where whater, ρ1 = 1.0 g/cm3 , some immiscible liquid, ρ2 = 4.0 g/cm3 .


Solving, gives us
h2 − h1 = 15 cm (0.132.2)
Let’ call the height of the water column on the left side of the tube, x1 . We get

FT
h2 − (x1 + 5) = 15
∴ h2 − x1 = 20

We expect the water column to go down on the left side of the tube as it goes up on the
right side of the tube; conservation of mass. So we infer the change in height on both
sides is 10 cm. We conclude that since the intial height is 20cm, then h2 = 30 cm and
x1 = 10 cm. So
h2 30
= =2 (0.132.3)
RA
h1 15
Answer: (C)

0.133 Sphere falling through a viscous liquid


Our sphere falls through a viscous liquid under gravity and experiences a drag force,
bv. The equation of motion can be expressed
D

ma = mg − bv (0.133.1)

We are also told that the buoyant force is negligible. Armed with this information, we
can analyze out choices and emiminate.

A This statement will be incorrect. We have been told to ignore the buoyant force,
which if was present, would act as a constant retarding force and slow our sphere
down and reduce its kinetic energy. INCORRECT

B This is also incorrect. In fact if you were to solve the above equation of motion, the
speed, and hence kinetic energy, would monotonically increase and approach
some terminal speed. It won’t go to zero. INCORRECT

C It may do this if it was shot out of a gun, but we were told that it is released from
rest. So it will not go past its terminal speed.

David S. Latchman ©2009


Moment of Inertia and Angular Velocity xci
D The terminal speed is the point when the force due to gravity is balanced by the
retarding force of the fluid. Setting ma = 0 in the above equation, we get
0 = mg − bv (0.133.2)
Solving for v yields,
mg
v= (0.133.3)
b
We see that our terminal velocity is dependent on both b and m. This choice is
INCORRECT
E From the above analysis, we choose this answer. CORRECT

Answer: (E)

FT
0.134 Moment of Inertia and Angular Velocity
The moment of inertia of an object is
N
X
I= mi r2i
i=1
RA
where ri is the distance from the point mass to the axis of rotation.
The moment of inertia about point A is found by finding the distances of each of the
three masses from that point. The distance between the mass, m and A is
`
r= √
3
Thus the moment of inertia is
!2
`
IA = 3m √ = m`2
3
D

The Moment of Inertia about B can be found by the Parallel Axis Theorem but it may be
simpler to use the formula above. As the axis of rotation is about B, we can ignore this
mass and find the distances of the other two masses from this point, which happens to
be `. Thus
IB = 2m`2
The rotational kinetic energy is
1
K = Iω2
2
So the ratio of the kinetic energies at fixed, ω becomes
KB IB 2m`2
= = =2
KA IA m`2
Answer: (B)

©2009 David S. Latchman


xcii GR9677 Exam Solutions
0.135 Quantum Angular Momentum

The probability is
32 + 22 13
P= = (0.135.1)
38 38
NOT FINISHED
Answer: (C)

0.136 Invariance Violations and the Non-conservation of


Parity

FT
Electromagnetic and strong interactions are invariant under parity transformations.
The only exception to this rule occurs in weak interactions, the β-decay bring one such
example. It had always been assumed that invariance was a “built-in” property of the
Universe but in the 1950s there seemed to be some puzzling experiments concerning
certain unstable particles called tau and theta mesons. The “tau-theta puzzle” was
solved in 1956 by T.D. Lee9 and C.N. Yang10 when they proposed the nonconservation of
RA
parity by the weak interaction. This hypothesis was confirmed experimentally through
the beta decay of Cobalt-60 in 1957 by C.S. Wu11 .

60
Co −−→ 60Ni + e – + ῡe

The cobalt source was chilled to a temperature of 0.01 K and placed in a magnetic
field. This polarized the nuclear spins in the direction of the magnetic field while the
low temperatures inhibited the thermal disordering of the aligned spins. When the
directions of the emitted electrons were measured, it was expected that there would be
D

equal numbers emitted parallel and anti-parallel to the magnetic field, but instead more
electrons were emitted in the direction opposite to the magnetic field. This observation
was interpreted as a violation of reflection symmetry.
Answer: (D)
9
Tsung-Dao Lee is a Chinese-born American physicist, well known for his work on parity violation,
the Lee Model, particle physics, relativistic heavy ion (RHIC) physics, nontopological solitons and
soliton stars. He and Chen-Ning Yang received the 1957 Nobel prize in physics for their work on parity
nonconservation of weak interactions.
10
Chen-Ning Franklin Yang is a Chinese-American physicist who worked on statistical mechanics
and particle physics. He and Tsung-dao Lee received the 1957 Nobel prize in physics for their work on
parity nonconservation of weak interactions.
11
Chien-Shiung Wu was a Chinese-American physicist. She worked on the Manhattan Project to
enrich uranium fuel and performed the experiments that disproved the conservation of parity. She
has been known as the“First Lady of Physics”, “Chinese Marie Curie” and “Madam Wu”. She died in
February 16, 1997

David S. Latchman ©2009


Wave function of Identical Fermions xciii
0.137 Wave function of Identical Fermions
The behavior of fermions are described by the Pauli Exclusion Principle, which states
that no two fermions may have the same quantum state. This is a results in the
anti-symmetry in the wave funtion.
Answer: (A)

0.138 Relativistic Collisions


We are told that no energy is radiated away, so it is conserved; all of it goes into the
composite mass. The relativistic energy is

FT
E = γmc2 (0.138.1)

Given that v = 3/5c


1 5
γ= q = (0.138.2)
v2 4
1− c2

So the energy of the lump of clay is


RA
5
E = γmc2 = mc2 (0.138.3)
4
The composite mass can be found by adding the energies of the two lumps of clay

ET = 2E
10
Mc2 = mc2
4
∴ M = 2.5m = 2.5 × 4 = 10 kg (0.138.4)

Answer: (D)
D

0.139 Relativistic Addition of Velocities


We recall that the relativistic addition formula
u+v
v0 = (0.139.1)
1 + uv
c2

where u = 0.3c and v = 0.6c. This becomes


0.9c 0.9 9
v0 = = c ≈ c = 0.75c (0.139.2)
1 + 0.18 1.18 12
Answer: (D)

©2009 David S. Latchman


xciv GR9677 Exam Solutions
0.140 Relativistic Energy and Momentum
The Relativistic Momentum and Energy equations are
p = γmv E = γmc2 (0.140.1)
We can determine the speed by dividing the relativistic momentum by the relativistic
energy equation to get
p γmv
=
E γmc2
v
= 2
c
5MeV/c v
∴ = 2
10MeV c

FT
5 v
= 2
10c c
1
⇒v= c (0.140.2)
2
Answer: (D)

0.141 Ionization Potential


RA
The Ionization Potential, or Ionization Energy, EI , is the energy required to remove one
mole of electrons from one mole of gaaseous atoms or ions. It is an indicator of the
reactivity of an element.
2
4
He The Helium atom is a noble gas and has filled outermost electron shells as well as
its electrons being close to the nucleus. It would be very difficult to ionize.
He = 1s2
D

7
14
N Nitrogen has two outermost electrons.

N = 1s2 , 2s2 , 2p6 , 2s2 , 2p2


8
16
O Oxygen has four outermost electrons.

O = 1s2 , 2s2 , 2p6 , 2s2 , 2p4


18
40
Ar Another noble gas, this has filled outermost electrons and is not reactive.
55
133
Cs We can see that Cs has a high atomic number and hence a lot of electrons. We
expect the outmost electrons to be far from the nucleus and hence the attraction
to be low. This will have a low ionization potential.

Answer: (E)

David S. Latchman ©2009


Photon Emission and a Singly Ionized He atom xcv
0.142 Photon Emission and a Singly Ionized He atom

The energy levels can be predicted by Bohr’s model of the Hydrogen atom. As a
Helium atom is more massive than Hydrogen, some corrections must be made to our
model and equation. The changes can be written

Z2 µ
En = − E0 (0.142.1)
n2 me

where Z is the atomic number, n is the energy level, E0 is the ground state energy level
of the Hydrogen atom and µ/me is the reduced mas correction factor.
The emitted photon can also be found through a similar correction

∆E =
hc
λe
= Z2
µ
me

FT 
 1
 2

 − 1  13.6

n f n2i 

As Helium’s mass is concentrated in the center, it’s reduced mass is close to unity12 .
(0.142.2)
RA
µ Z
= ≈1 (0.142.3)
me Z + me

Plugging in the values we know into eq. (0.142.2), we get


 
6.63 × 10−34 × 3 × 108  1 1 
= 22
13.6  − 
 n2 42  (0.142.4)
470 × 10−9 × 1.60 × 10−19

f

After some fudging and estimation we get


D

6.63 × 3 19
× 102 ≈ × 102
470 × 1.6 470 × 1.6
20
≈ × 102
750
= 0.026 × 102 eV (0.142.5)

and
2.6 1

22 · 13.6 20

12
It is helpful to know that in the case of atoms, the reduced mass will be close to unity and can be
ignored from calculation. In the case of smaller bodies, e.g. positronium, this correction factor can not
be ignored.

©2009 David S. Latchman


xcvi GR9677 Exam Solutions
Solving for n f , gives13
1 1 1
= 2 − 2
20 n f 4
1 1 1 9
2
= + =
nf 20 16 80
1

9
∴ nf = 3 (0.142.6)

Now we can calculate the energy level at n = 3 from eq. (0.142.1), which gives,

22
E3 = − · 13.6
32

FT
= −6.0 eV (0.142.7)

We get E f = −6.0 eV and n f = 3. This corresponds to (A).


Answer: (A)

0.143 Selection Rules


RA
NOT FINISHED
Answer: A

0.144 Photoelectric Effect


This question deals with the photoelectric effect which is essentially an energy conser-
vation equation. Energy of a photon strikes a metal plate and raises the electrons to
D

where they can leave the surface. Any extra energy is then put into the kinetic energy
of the electron. The photoelectric equation is

h f = eVs + K (0.144.1)

As our choices are in electron-volts, our equation becomes


hc
= eVs + K (0.144.2)

where K is the kinetic energy of our photoelectrons. Plugging in the values we were
given and solving for K, we get
K = 0.2 eV (0.144.3)
Answer: (B)
13
As n f = 3 is only in choice (A), we can forego any further calculation and choose this one.

David S. Latchman ©2009


Stoke’s Theorem xcvii
0.145 Stoke’s Theorem
NOT FINISHED
Answer: (C)

0.146 1-D Motion


A particle moves with the velocity

v(x) = βx−n (0.146.1)

To find the acceleration, a(x), we use the chain rule

FT
dv dx dv
a(x) = = ·
dx dt dx
dv
=v· (0.146.2)
dx
Differentiating v(x) with respect to x gives

dv
= −nβx−n−1
RA
dx
Thus, our acceleration, a(x), becomes

a(x) = βx−n · −nβx−n−1


= −nβ2 x−2n−1 (0.146.3)

Answer: (A)

0.147 High Pass Filter


D

Capacitors and Inductors are active components; their impedances vary with the fre-
quency of voltage unlike an ohmic resistor whose resitance is pretty much the same no
matter what. The impedances for capacitors and inductors are

1
XC = XL = ωL
ωC
We see that in the case of capacitors, there is an inverse relationship with frequency
and a linear one for inductors. Simply put, at high frequencies capacitors have low
impedances and inductors have high inductances.
NOT FINSIHED
Answer: (E)

©2009 David S. Latchman


xcviii GR9677 Exam Solutions
0.148 Generators and Faraday’s Law
The induced EMF in the loop follows Faraday’s Law

E =−
dt
In this case, the magnetic field, B, is constant and the Cross Sectional Area, A, through
which the magnetic field acts changes. Thus the above equation becomes
dA
E0 sin ωt = −B (0.148.1)
dt
Let’s say that at t = 0 the loop is face on with the magnetic field,

A = πR2 cos ωt (0.148.2)

Substituting this into eq. (0.148.1) gives

E0 sin ωt = −B ·

FT
dA
dt
= −B · −ωπR2 sin ωt
= ωBπR2 sin ωt


(0.148.3)
RA
Solving for ω gives
E0
ω= (0.148.4)
BπR2
Answer: (C)

0.149 Faraday’s Law and a Wire wound about a Rotating


Cylinder
D

The induced EMF of our system can be found from Faraday’s Law, where

E =− (0.149.1)
dt
Here the flux changes because the number of loops enclosing the field increases, so

Φ = NBA (0.149.2)

Substituting this into Faraday’s Equation we get


dN
E = BA
dt
= BπR2 N (0.149.3)

Answer: (C)

David S. Latchman ©2009


Speed of π+ mesons in a laboratory xcix
+
0.150 Speed of π mesons in a laboratory
As the π+ meson travels through our laboratory and past the detectors, its half life
is time dilated in our laboratory’s rest frame. We can also look at things in the π+
meson’s rest frame. In this case, the distance it travels will be length contracted in it
rest frame. The speed of our π+ mesons is the length divided by the time dilation in the
laboratory’s rest frame or the length contraction in the π+ meson’s rest frame divided
by its half life. In either case, we get
r
L v2
v= 1− 2
T1/2 c
Factorizing we get  
2 
L L2

FT
v2 1 + 2 2  = 2

(0.150.1)
 
c T1/2 T1/2
We see that
L
= 6 × 108
T1/2
Plugging this into eq. (0.150.1), the speed in terms of c

v2 36 36
 
1+ =
RA
c 2 9 9
v2
(5) = 4
c2
2
⇒v= √ c (0.150.2)
5
Answer: (C)

0.151 Transformation of Electric Field


D

NOT FINSHED
Answer: (C)

0.152 The Space-Time Interval


We have two events, in the S-frame,

S1 (x1 , t) and S2 (x2 , t)

In the S0 -frame, the co-ordintes are

S01 (x01 , t01 ) and S02 (x02 , t02 )

©2009 David S. Latchman


c GR9677 Exam Solutions
The Space-Time Interval in the S-frame

∆S = ∆x2 = 3c minutes (0.152.1)

In the S0 -frame, the Space-Time Interval is

∆S0 = ∆x02 − c2 ∆t02 = 5c minutes (0.152.2)

The Space-Time Interval is invariant across frames, so eq. (0.152.1) is equal to eq. (0.152.2)

(3c)2 = (5c)2 − c2 ∆t2


⇒ ∆t = 4 minutes (0.152.3)

Answer: (C)

0.153

FT
Wavefunction of the Particle in an Infinte Well

The wave function has zero probability density in the middle for even wave functions,
n = 2, 4, 6, · · · .
Answer: (B)
RA
0.154 Spherical Harmonics of the Wave Function

NOT FINSIHED
Answer: (C)
D

0.155 Decay of the Positronium Atom

NOT FINSHED
Answer: (C)

0.156 Polarized Electromagnetic Waves I

We are given an electromagnetic wave that is the superposition of two independent


orthogonal plane waves where

E = x̂E1 exp [i (kz − ωt)] + ŷE2 exp [i (kz − ωt + π)] (0.156.1)

David S. Latchman ©2009


Polarized Electromagnetic Waves II ci
As we are looking at the real components and E1 = E2 , we have

E = <(E1 eikz · e−iωt )x̂ + <(E1 eikz · e−iωt · e−iπ )ŷ


= <(E1 eikz · e−iωt )x̂ − <(E1 eikz · e−iωt )ŷ

We see that the x̂ and ŷ vectors have the same magnitude but opposite sign; they are
both out of phase with each other. This would describe a trajectory that is 135°to the
x-axis.
Answer: (B)

0.157 Polarized Electromagnetic Waves II

FT
NOT FINISHED
Answer: (A)

0.158 Total Internal Reflection


RA
Total internal reflectance will occur when the incident beam, reaches a critical angle,
θi , such that the refracted angle just skims along the water’s surface, θr = 90◦ . We can
find this critical angle using Snell’s Law

n2 sin θi = n1 sin 90

where n2 = 1.33 and n1 = 1, we have

1 3
sin θ2 == (0.158.1)
1.33 4
D


We know that sin 30° = 1/2 and sin 60° = 3 /2, so 30° < θ < 60°.
Answer: (C)

0.159 Single Slit Diffraction

For a single slit, diffraction maxima can be found from the formula

a sin θ = mλ (0.159.1)

where a is the slit width, θ is the angle between the minimum and the central maximum,
and m is the diffraction order. As θ is small and solving doe d, we can approximate the

©2009 David S. Latchman


cii GR9677 Exam Solutions
above equation to

λ
d=
θ
400 × 10−9
=
4 × 10−3
= 0.1 × 10−3 m (0.159.2)

Answer: (C)

0.160 The Optical Telescope


The magnification of the optical telescope can be found from the focal length of the

FT
eyepiece, fe and the objective, fo . Thus

fo
M= = 10 (0.160.1)
fe

The focal length of the objective is

fe = 10 × 1.5 = 15 cm (0.160.2)
RA
To achieve this magnification, the lens must be placed in a position where the focal
length of the eyepiece meets the focal length of the objective. Thus

D = fe + fo = 15.0 + 1.5 = 16.5 cm (0.160.3)

Answer: (E)

0.161 Pulsed Lasers


D

Lasers operating in pulsed mode delivers more energy in a short space of time as
opposed to delivering the same energy over a longer period of time in a continuous
mode. While there are several methods to achieve a pulsed mode, beyond what is
needed to answer this question, we can determine the number of photons delivered
by such a device. The energy of a photon is

hc
E = hf = (0.161.1)
λ
The power is the energy delivered in one second. So for a 10kW laser, the total energy
in 10−15 seconds is

EL = Pt = 10 × 103 × 10−15
= 10 × 10−12 J (0.161.2)

David S. Latchman ©2009


Relativistic Doppler Shift ciii
So the total number of photons is

EL
n=
E
10 × 10−12 × λ
=
hc
10 × 10−12 × 600 × 10−9
=
6.63 × 10−34 × 3 × 108
10 × 600
= × 106 ≈ 3 × 108 (0.161.3)
6.63 × 3
Answer: (B)

FT
0.162 Relativistic Doppler Shift
The relativistic doppler shift is
s
λo fs 1+β
= = (0.162.1)
λs fo 1−β
RA
The redshift is calculated to be
λo − λs fs − fo
z= = (0.162.2)
λs fo

we can rewrite eq. (0.162.2) as


s
1+β
z= −1 (0.162.3)
1−β
In the non-relativistic limit, v << c, we can approximate eq. (0.162.3) to
D

v
z≈β= (0.162.4)
c
This, equating eq. (0.162.2) and eq. (0.162.4), we see that

∆f
v= c (0.162.5)
f

Substituting the values given into eq. (0.162.5), we have

0.9 × 10−12
v= × 3 × 108
122 × 10−9
≈ 2.2 m s−1 (0.162.6)

Answer: (B)

©2009 David S. Latchman


civ GR9677 Exam Solutions
0.163 Gauss’ Law, the Electric Field and Uneven Charge
Distribution

We can find the electric field in a non-conducting sphere by using Gauss’ Law
I
Qenclosed
E · dA = (0.163.1)
0

The enclosed charge can be found from the charge density, which is

Enclosed Charge
ρ=
Enclosed Volume
q
=

FT
(0.163.2)
4 3
πr
3

We can find the enclosed charge by integrating within 0 to R/2. The charge density is

dq
ρ=
dV
dq
=
RA
(0.163.3)
4πr2 dr
∴ dq = ρ4πr2 dr
= 4πAr4 dr (0.163.4)

Gauss’ Law becomes


I
qenclosed
E · dA =
0
Z R
  2 dq
E 4πr =
2
D

0 0
Z R2
4πA
= r4 dr
0 0
R
4πA r5 2
= (0.163.5)
0 5 0

R
A r3 2
∴E= (0.163.6)
0 5 0

Solving gives
AR3
E= (0.163.7)
400
Answer: (B)

David S. Latchman ©2009


Capacitors in Parallel cv
0.164 Capacitors in Parallel

We initially charge both of our capacitors in parallel across a 5.0V battery. The charge
stored on each capacitor is
Q1 = C1 V Q2 = C2 V

where V = 5.0 V, C1 = 1µF and C2 = 2µF.


The battery is then disconnected and the plates of opposite charges are connected
to each other. This results in the excess charges cancelling each other out and then
redistributing themselves until the potential across the new configuration is the same.
The charge left after this configuration is

FT
QA = Q2 − Q1 (0.164.1)

The charges on each capacitor becomes

Q1A + Q2A = QA (0.164.2)

where
Q1A = C1 V f Q2A = C2 V f
RA
and (0.164.3)

Solving for V f gives us


(C2 − C1 )V 5
Vf = = = 1.67 V (0.164.4)
C1 + C2 3

Answer: (C)

0.165 Standard Model


D

NOT FINSIHED
Answer: (A)

0.166 Nuclear Binding Energy

Typically a heavy nucleus contains ∼ 200 nucleons. The energy liberated would be the
difference in the binding energies 1 MeV ×200.
Answer: (C)

©2009 David S. Latchman


cvi GR9677 Exam Solutions
0.167 Work done by a man jumping off a boat
The work the man does is the sum of the kinetic energies of both the boat and himself.
We can find the speeds of the man and the boat because momentum is conserved.
mu = Mv
Mv
u= (0.167.1)
m
The total energy of the system, and hence the work the man does in jumping off the
boat is
1 1
W = mu2 + Mv2
2  2
1 2 M

= Mv +1 (0.167.2)

FT
2 m
Answer: (D)

0.168 Orbits and Gravitational Potential


For an attractive potential, such as we would expect for the Gravitational Potential, we
RA
have

Orbit Total Energy


Ellipse E<0
Parabola E=0
Hyperbola E>0

where E is the total energy of the system; potential and kinetic energy. As the potential
D

energy of the system remains unchanged, the only difference is the is the kinetic energy,
the orbit will be hyperbolic.
When the spacecraft has the same speed as Jupiter, the orbit will be locked and will be
elliptical. If the gravitational potential energy was equal to the kinetic energy, the orbit
will no longer be locked and will be parabolic. We assume that the huge difference will
cause the orbit to be hyperbolic.
Answer: (E)

0.169 Schwartzchild Radius


Any mass can become a black hole if it is compressed beyond its Schwarzschild Radius.
Beyond this size, light will be unable to escape from it’s surface or if a light beam were

David S. Latchman ©2009


Lagrangian of a Bead on a Rod cvii
to be trapped within this radius, it will be unable to escape. The Schwarzschild Radius
can be derive by putting the Gravitational Potential Energy equal to a mass of kinetic
energy travelling at light speed.

GMm 1 2
= mc (0.169.1)
R 2
Solving for R yields
2GM
R= (0.169.2)
c2
Plugging in the values given, we get

2 × 6.67 × 10−11 × 5.98 × 102 4


R=
(3 × 108 )2

FT
Our indices indicate we will get an answer in the order ≈ 10−3 meters.
Answer: (C)

0.170 Lagrangian of a Bead on a Rod


RA
The Lagrangian of a system is defined

L=T−V (0.170.1)

The rod can move about the length of the rod, s and in circular motion along a radius
of s sin θ. The Lagrangian of this system becomes

1 1
L = mṡ2 + m(s sin θ)2 ω2 − mgs cos θ (0.170.2)
2 2
D

Answer: (E)

0.171 Ampere’s Law

We can use Ampere’s Law to tell us the magnetic field at point A.


I
B · ds = µ0 Ienclosed (0.171.1)

The point A is midway between the center of the two cylinders and as the currents are
in opposite directions, thier magnetic fields at A point in the +y-direction. We can use
the right hand grip rule to determine this. This leaves us with choices (A) or (B).

©2009 David S. Latchman


cviii GR9677 Exam Solutions
The current density, J is

I
J=
Area
I
= 2 (0.171.2)
πr
We draw an Amperian loop of radius, r = d/2, thus the magnetic field becomes,
I
B · ds = µ0 Ienclosed
 
B · (2πr) = µ0 J πr2
µ0 πJr
B=

FT
µ0 πJ d
= (0.171.3)
2π 2
We expect B⊗ to be the same, thus

B = B + B⊗
µ 
0
= πdJ (0.171.4)

RA
Answer: (A)

0.172 Larmor Formula


The Larmor Formula is used to calculate total power radiated by an accelerating non-
relativistic point charge.
e2 a2
P= (0.172.1)
6π0 c3
D

where a is the acceleration. For particles A & B, we are given

A B
Charge qa = q qb = 2q
Mass ma = m mb = m/2
Velocity va = v vb = 3v
Acceleration aa = a ab = 4a

From the above, we see that

PA ∝ q2 a2 PB = (2q)2 (4a2 )

David S. Latchman ©2009


The Oscilloscope and Electron Deflection cix
Thus
PB (2q)2 (4a2 )
=
PA q2 a2
= 64 (0.172.2)

Answer: (D)

0.173 The Oscilloscope and Electron Deflection


As the electron passes through the deflection plates, the electric field, E exerts a force
on the charge, pulling it up. Ignoring gravitational effects, the electric force is

FT
qV
Fe = qE = = ma y (0.173.1)
d
The time it takes to traverse this distance is
L
t= (0.173.2)
v
The deflection angle, θ is determined by
vy
RA
tan θ = (0.173.3)
vx
Now

vy = ayt
qV
= t
me d
qV L
= (0.173.4)
me d v
D

The the horizontal speed is v The angle of deflection becomes


qV L
tan θ = md v
v
qVL
= (0.173.5)
mdv2
Answer: (A)

0.174 Negative Feedback


All amplifiers exhibit non-linear behavior of some sort. Negative feedback seeks to
correct some of these effects by sending some of the output back and subtracting it from

©2009 David S. Latchman


cx GR9677 Exam Solutions
the input. This results in a decrease in gain. This tradeoff og gain improves linearity
and hence the stability of the amplifier. This also allows for increased bandwidth
response and decreased distortion.
Answer: (A)

0.175 Adiabatic Work of an Ideal Gas


The adiabatic condition states that
PV γ = C (0.175.1)
The work done by an ideal gas is
Z

FT
W= PdV (0.175.2)

Substituting the adiabatic condition into the work equation yields


Z Vf
dV
W=C γ
Vi V
V
V −γ+1 f
=C
1 + γ Vi

RA
C h −γ+1 −γ+1
i
= Vf − Vi (0.175.3)
1−γ
The adiabatic condition is
γ γ
C = PV γ = Pi Vi = P f V f (0.175.4)
Substituting this into eq. (0.175.3), we get
−γ+1 −γ+1
CV f − CVi
W=
1−γ
D

γ −γ+1 γ −γ+1
Pf Vf Vf − Pf Vf Vf
=
1−γ
P f V f − Pi Vi
= (0.175.5)
1−γ
Answer: (C)

0.176 Change in Entrophy of Two Bodies


The change in entrophy of a system is
dQ
dS = (0.176.1)
T

David S. Latchman ©2009


Double Pane Windows cxi
The change in heat of a system is
dQ = nCdT (0.176.2)
Substituting this into the above equation, we get
Z Tf
dT
dS = mC
Ti T
" #
Tf
= mC ln (0.176.3)
Ti
We are told that the two bodies are brought together and they are in thermal isolation.
This means that heat is not absorbed from or lost to the environment, only transferred
between the two bodies. Thus
Heat Lost by Body A = Heat Gained by Body B

FT
   
mC 500 − T f = mc T f − 100
∴ T f = 300 K
The Total Change in Entrophy is the sum of the entrophy changes of bodies A and B.
Thus
dS = dSA + dSB
300 300
   
= mC ln + mC ln
500 100
RA
9
 
= mC ln (0.176.4)
5
Answer: (B)

0.177 Double Pane Windows


The rate of heat transer is proportional to the temperature difference across the body,
is stated in Newton’s Law of Cooling.
D

dQ
= kxdT (0.177.1)
dt
where k is the thermal conductivity, x is the thickness of the material and dT is the
temperature difference across the material.
For Window A
dQA
PA = = 0.8 × 4 × 10−3 dT (0.177.2)
dt
For Window B
dQB
PB = = 0.025 × 2 × 10−3 dT (0.177.3)
dt
The ratio of heat flow is
Pa 0.8 × 4 × 10−3 dT
= = 16 (0.177.4)
PB 0.025 × 2 × 10−3 dT
Answer: (D)

©2009 David S. Latchman


cxii GR9677 Exam Solutions
0.178 Gaussian Wave Packets
We have a Gaussian wave packet travelling through free space. We can best think of
this as an infinite sum of a bunch of waves initially travelling together. Based on what
we know, we can eliminate the choices given.

I The average momentum of the wave packet can not be zero as p = ~k. As we
have a whole bunch of wave numbers present, the average can not be zero.
INCORRECT
II Our wave packet contains a bunch of waves travelling together each with a differnt
wave vector, k. The speed of propagation of these individual wave vectors is
defined by the group velocity, v g = dω/dk. So some waves will travel, some
slower than others. As a result of these different travelling rates our wave packet

FT
becomes spread out or ‘dispersed’. This is the basis of our dispersion relation,
ω(k), relative to the center of the wave packet. CORRECT
III As we expect the wave packet to spread out, as shown above, the amplitude will
decrease over time. The energy that was concentrated in this packet gets spread
out or dispersed. INCORRECT
IV This is true. This statement is the Uncertainty Principle and comes from Fourier
Analysis. CORRECT
RA
We see that choices II and IV are CORRECT.
Answer: (B)

0.179 Angular Momentum Spin Operators


NOT FINISHED
Answer: (D)
D

0.180 Semiconductors and Impurity Atoms


NOT FINISHED
Answer: (B)

0.181 Specific Heat of an Ideal Diatomic Gas


The formula for finding the molar heat capacity at constant volume can be found by
!
f
cv = R (0.181.1)
2

David S. Latchman ©2009


Transmission of a Wave cxiii
where f is the number of degrees of freedom. At very low temperatures, there are only
three translational degrees of freedom; there are no rotational degrees of freedom in
this case. At very high temperatures, we have three translational degrees of freedom,
two rotational and two vibrational, giving a total of seven in all.

For Low Temperatures For High Temperatures


Translational 3 Translational 3
Rotational 0 Rotational 2
Vibrational 0 Vibrational 2
Total( f ) 3 Total( f ) 7

Table 0.181.1: Table of degrees of freedom of a Diatomic atom

and at very high temperatures, FT


We see that in the case of very low temperatures,

7
3
cvl = R
2

cvh = R
(0.181.2)

(0.181.3)
RA
2
Thus, the ratio of molar heat capacity at constant volume at very high temperatures to
that at very low temperatures is
7
cvh 2
R
= 3
cvl 2
R
7
= (0.181.4)
3
Answer: (D)
D

0.182 Transmission of a Wave

NOT FINISHED
Answer: (C)

0.183 Piano Tuning & Beats

The D2 note has a frequency of 73.416 Hz and the A4 note has a frequency of 440.000 Hz.
Beats are produced when the two frequencies are close to each other; if they were the

©2009 David S. Latchman


cxiv GR9677 Exam Solutions
same, there would be no beat frequency. So we can determine the D2 note where this
happens by setting the beat frequency to zero.

440.000 − n(73.416) = 0 (0.183.1)

This works out to be


440.000
n=
73.416
440
≈ =6 (0.183.2)
72
Thus the closest harmonic will be the 6th one. As we expect this to be very close to the
A4 frequency, the number of beats will be small or close to zero. Answer (B) fits this.14
Answer: (B)

0.184 Thin Films

FT
As light moves from the glass to air interface, it is partially reflected and partially
transmitted. There is no phase change when the light is refected. In the case of the
transmitted wave, when it reaches the air-glass interface, there is a change in phase of
RA
the reflected beam. Thus the condition for destructive interference is
1
 
2L = n + λ (0.184.1)
2

where L is the thickness of the air film and n = 0, 1, 2 is the interference mode. Thus
2n + 1
 
L= λ (0.184.2)
4
We get
λ
D

L0 = = 122 nm (0.184.3)
4

L1 = = 366 nm (0.184.4)
4

L2 = = 610 nm (0.184.5)
4
Answer: (E)
14
Incidentally, you can multiply the D2 frequency by six to determine the harmonic. This turns out to
be
73.416 × 6 = 440.496 Hz (0.183.3)
Subtracting this from the A2 frequency gives

440.496 − 440.000 = 0.496 Hz (0.183.4)

David S. Latchman ©2009


Mass moving on rippled surface cxv
0.185 Mass moving on rippled surface
For the mass to stay on the rippled surface, the particle’s horizontal velocity should
not be so great that it flies off the track. So as it falls, it must hug the track. The time
for the particle to fall from the top of the track to the bottom is
1 2
2d = gt (0.185.1)
2
where s
4d
t= (0.185.2)
g
We could also have said,
2π 1

FT
 
d − d cos k · = gt2
2k 2
1
∴ 2d = gt2
2
but this is the same as eq. (0.185.2).
To stay on the track, the particle must cover a horizontal distance of x = π/k in the
same time. Thus the horizontal speed, v, is
RA
x π g 1/2
 
v= =
t k 4d
π
 g 1/2
= · 2 (0.185.3)
2 kd
Any speed greater that 0.185.3 would result in the particle flying off the track. So
r
g
v≤ (0.185.4)
k2 d

Answer: (D)
D

0.186 Normal Modes and Couples Oscillators


NOT FINISHED
Answer: (D)

0.187 Waves
NOT FINSIHED
Answer: (B)

©2009 David S. Latchman


cxvi GR9677 Exam Solutions
0.188 Charged Particles in E&M Fields

NOT FINISHED
Answer: (B)

0.189 Rotation of Charged Pith Balls in a Collapsing Mag-


netic Field

As the magnetic field, B, collapses, it indices a clockwise electric field, E, that causes
the charged pith balls to rotate. The electric field can be found from Faraday’s Law

FT
∂Φ
I
E · d` = − (0.189.1)
∂S ∂t

The magnetic flux covers an area, A, of

A = πR2 (0.189.2)

and the electric field makes a loop of circumference


RA
d` = πd (0.189.3)

Substituting eqs. (0.189.2) and (0.189.3) into eq. (0.189.1) gives us

dB
E · πd = πR2 (0.189.4)
dt
Solving for E, gives us
R2 dB
E=
D

(0.189.5)
d dt
The question answers ask for the angular momentum, we recall that the torque is

τ=r×F (0.189.6)

where F = qE. Substituting eq. (0.189.5) into eq. (0.189.6)

dL dB
= qR2 (0.189.7)
dt dt
So it follows that
L = qR2 B (0.189.8)

Answer: (A)

David S. Latchman ©2009


Coaxial Cable cxvii
0.190 Coaxial Cable
We expect there to be no magnetic field outside, r > c, the coaxial cable. Choices (D)
and (E) make no sense. So choice (B) is our best one left.
We can also use Ampere’s Law to show what the magnetic induction will look like as
we ove away from the center. We recall Ampere’s Law
I
B · d` = µ0 Ienclosed (0.190.1)

0 < r < a The current enclosed by our Amperian Loop is


πr2
Ienclosed = I (0.190.2)
πa2

FT
Ampere’s Law shows us
πr2
B (2πr) = µ0 I
πa2
µ0 I r
B(0<r<a) = (0.190.3)
2π a2
We see the magnetic induction will increase linearly with respect to r.
a < r < b Within the shaded region, the enclosed current is, Ienclosed = I. Ampere’s Law
RA
becomes
B (2πr) = µ0 I
µ0 I
B(a<r<b) = (0.190.4)
2π r
The magnetic induction decreases inversely with respect to r.
b < r < c The area of the outer sheath is
 
A = π c2 − b 2 (0.190.5)
D

The enclosed current will be


πr2 − πb2
" #
Ienclosed =I−I
πc2 − πb2
" 2 #
c − r2
=I 2 (0.190.6)
c − b2
Ampere’s Law becomes
" #
c2 − r2
B (2πr) = µ0 I 2
c − b2
µ0 I c − r2
" 2 #
B(b<r<c) = (0.190.7)
2πr c2 − b2
This will fall to an inversely with respect to r.

©2009 David S. Latchman


cxviii GR9677 Exam Solutions
r > c We see that Ienclosed = 0, so from Ampere’s Law

B(r>c) = 0 (0.190.8)

The graph shown in (B) fits this15 .


Answer: (B)

0.191 Charged Particles in E&M Fields

A seemingly difficult question but it really is not16 . All we need to turn to is Pythagoras

FT
Theorem and relate the centripetal force to the Lorentz Force Law.
From the Lorentz Force Law, we see that

mv2
= Bqv (0.191.1)
r

We can simplify this to read,


RA
p = Bqr (0.191.2)

We know B and q. We can determine r in terms of s and ` by using Pythagoras Theorem.

r2 = `2 + (r − s)2 (0.191.3)

Solving for r, gives us


1 `2
!
r= +s (0.191.4)
2 s
D

As s << `, eq. (0.191.4) becomes


1 `2
r= (0.191.5)
2 s
Substituting eq. (0.191.5) into eq. (0.191.2), gives us

Bq`2
(0.191.6)
2s

Answer: (D)
15
This is one of the reasons we use coaxial cables to transmit signals. No external magnetic field from
our signals means that we can, theoretically, eliminate electromagnetic interference.
16
Draw Diagrams

David S. Latchman ©2009


THIS ITEM WAS NOT SCORED cxix
0.192 THIS ITEM WAS NOT SCORED

0.193 The Second Law of Thermodynamics


This question deals with the Second Lar of Thermodynamics, which states that the
entrophy of an isloated system, which is not in equilibrium, will increase over time,
reaching a maximum at equilibrium. An alternative but equivalent statement is the
Clausius statement, “Heat can not flow from cold to hot without work input”. Thusit
would be impossible to attain the 900 K temperature the experimenter needs with a
simple lens.
Answer: (E)

0.194 Small Oscillations

FT
We are given a one dimensional potential function

V(x) = −ax2 + bx4 (0.194.1)

We can find the points of stability by differentiating the above equation to get and
RA
setting it to zero.
dV
= −2ax + 4bx3 = 0 (0.194.2)
dx
we see that x = 0; a/2b. Taking the second differential gives us the mass’s spring
constant, k
d2 V
k= = −2a + 12bx2 (0.194.3)
dx2
We can also use this to find the minimum and maximum points of inflection in our
potential graph. We see that
D

d2 V(0) d2 V(a/2b)
k= = −2a k= = 4a
dx2 dx2
We see that when x = a/2b, we are at a minima and hence at a point of stable equilibrium.
We can now find the angular frequency
r
k
ω=
m
r
4a
=
m
r
a
=2 (0.194.4)
m

Answer: (D)

©2009 David S. Latchman


cxx GR9677 Exam Solutions
0.195 Period of Mass in Potential

The total period of our mass will be the time it takes to return to the same point, say
the origin, as it moves through the two potentials.

for x < 0 The potential is


1
V = k2 (0.195.1)
2
The spring constant, k, is
d2 V
k= =k (0.195.2)
dx2
Thus the period of oscillation would be

FT
r
m
T = 2π (0.195.3)
k

But this represents the period of the mass could also swing from x > 0. As a
result, our period would just be half this. So
r
m
T(x<0) = π (0.195.4)
RA
k

for x > 0 The potential is


V = mgx (0.195.5)
We may recognize this as the gravitational potential energy. In this case, the
”period” would be the time for the mass to return the origin. This would simply
be
1
s = v0 t − gt2 (0.195.6)
2
where s = 0. Solving for t, we get t = 0 and t = 2v0 /g We can solve this in terms
D

of the total energy, E of the mass. The energy is E = 1/2mv20 . Our period works
out to be s
2E
T(x>0) = 2 (0.195.7)
mg2

Thus our total period is

T = T(x<0) + T(x>0)
r s
m 2E
=π +2 (0.195.8)
k mg2

Answer: (D)

David S. Latchman ©2009


Internal Energy cxxi
0.196 Internal Energy
NOT FINISHED
Answer: (D)

0.197 Specific Heat of a Super Conductor


Superconductors experience an increase in their specific heat, Cv , at the transition
temperature, Tc . At high temperatures, the specific heat falls linearly but at low
temperatures, the specific heat falls below the linear dependence of a degenrate Fermi
gas and is proportional to exp(−k/T). If the superconductor is placed in a magnetic
field that exceeds the critical field strength, B > Bc , the specific heat reverts to the

FT
normal linear behavior.
We see both the linear dependence and exponential decay below a certain value in
Choice:(E).
Answer: (E)

0.198 Pair Production


RA
We are given the equation of pair production
γ → e− + e+ (0.198.1)
Pair production occurs when a γ ray of high energy is absorbed in the vincinity of
an atomic nucleus and particles are created from the absorbed photon’s energy. This
takes place in the Coulomb field of the nucleus; the nucleus acts as a massive body to
ensure the conservation of momentum and energy. The nucleus is an essential part of
this process; if the photon could spontaneously decay into an electron-positron pair in
D

empty space, a Lorentz frame could be found where the electron and positron have
equal and opposite momenta and the photon will be at rest. This is a clear violation of
the principles of Special Relativity. We choose (A).
Based on what we know, we can determine the maximum wavelength as a matter or
interest. As the nucleus is massive, we will ignore its recoil and consider that all of the
photon’s energy goes into electron-positron creation and the particles’ kinetic energy.
hυ = E− + E+
= K− + me c2 + K+ + me c2
   

= K− + K+ + 2me c2 (0.198.2)
where K− and K+ are the kinetic energies of the electron and positron respectively. Thus
the minimum energy needed to initiate this process is
hυmin = 2me c2 (0.198.3)

©2009 David S. Latchman


cxxii GR9677 Exam Solutions
The wavelength of this photon is

h
λmax = (0.198.4)
2me c2

which happens to be half the Compton wavelength.


Answer: (A)

0.199 Probability Current Density

The probability current density is defined as17

FT
∂Ψ ∂Ψ∗
!
~
j(x, t) − Ψ∗ − Ψ (0.199.1)
2im ∂x ∂x

Thus given the wavefunction

Ψ(x, t) = eiωt α cos kx + β sin kx


 
(0.199.2)

Its complex conjugate is


RA
Ψ∗ (x, t) = e−iωt α∗ cos kx + β∗ sin kx
 
(0.199.3)

Differentiating the above two equations yields


= eiωt −kα sin kx + kβ cos kx
 
(0.199.4)
dx

dΨ∗
= e−iωt −kα∗ sin kx + kβ∗ cos kx
 
(0.199.5)
dx
D

Substituting eqs. (0.199.2) to (0.199.5) into eq. (0.199.1) yields

k~
j(x, t) = α∗ β − β∗ α

(0.199.6)
2im
Answer: (E)

0.200 Quantum Harmonic Oscillator Energy Levels


Given the potential
1
V(x) = mω2 x2 (0.200.1)
2
17
Add derivation in section.

David S. Latchman ©2009


Three Level LASER and Metastable States cxxiii
Solving the Schrödinger’s Equation for this potential

~2 d2 ψ 1
− + mω2 x2 ψ = Eψ (0.200.2)
2m dx2 2

leaves us with
 14
mω 1

Hn e−ξ /2
2
ψn = √ (0.200.3)
π~ 2n n!

where H( n) are Hermite polynomials and with energy levels of18

FT
1
 
En = n + ~ω (0.200.4)
2

But we have placed an infinitely large barrier, V = ∞ at x ≤ 0. This serves to constrain


ψn (0) = 0 at x = 0. This occurs at n = 1, 3, 5, . . . or rather only odd values of n are
allowed. Thus
3 5 7 11
En = ~ω, ~ω, ~ω, ~ω, . . .
RA
(0.200.5)
2 2 2 2

Answer: (D)

0.201 Three Level LASER and Metastable States


D

Metastability describes a state of delicate equilibrium. Such a system is in a state of


equilibrium but is susceptible to fall into a lower-energy state with a slight interaction.

The laser operation depends on an active medium of atoms whose energy states can
be populated selectively by radiative means.19 In the three-level laser, a discharge
of some sort raises atoms from the ground state, E1 , to a higher state, E3 . A rapid
spontaneous decay then occurs, bringing the excited atoms down to the E2 state. This
state is metastabe as it inhibits spontaneous decay back down to the ground state. It
needs an incident photon of energy hυ = E2 −E1 to stimulate the desired laser transition
back down to the ground state, E1 .

18
Add wavefunctions here
19
Add laser explanation in one of the sections.

©2009 David S. Latchman


cxxiv GR9677 Exam Solutions
E3

Pumping Transition
E2 Metastable Level

E1

Figure 0.201.1: Three Level Laser

FT
The Ruby Laser is an example of a three-level laser. Green light from a flash lamp
pumps the chromium ions to an excited level and non-radiative de-excitation promptly
brings the ions to a long lived metastable state. Stimulated emission then follows,
generating a coherent beam of red light of 694 nm.
Answer: (B)
RA
0.202 Quantum Oscillator – Raising and Lowering Oper-
ators
We are given the lowering operator
r !
mω0 p̂
â = x̂ + i (0.202.1)
2~ mω0

The raising operator is given by


D

r !
mω0 p̂
â =

x̂ − i (0.202.2)
2~ mω0

So we immediately see that


↠, â (0.202.3)
which we see from Choice: III. But what about the other two choices? In reality we
only have to prove/disprove Choice: I but we will go through both.
We know that â is not Hermitian from eq. (0.202.3) and hence not observable. Hence
we can eliminate Choice: II.
This affects Choice: I as â is not observable and hence won’t commute with the
Hamiltonian, H. We eliminate Choice: I.
Answer: (C)

David S. Latchman ©2009


Constants & Important Equations

.1 Constants

Constant
Speed of light in a vacuum
Gravitational Constant
Rest Mass of the electron
Avogadro’s Number
Universal Gas Constant
FT
Symbol

G
c

me
NA
R
Value
2.99 × 108 m/s
6.67 × 10−11 m3 /kg.s2
9.11 × 10−31 kg
6.02 × 1023 mol-1
8.31 J/mol.K
1.38 × 10−23 J/K
RA
Boltzmann’s Constant k
Electron charge e 1.60 × 10−9 C
Permitivitty of Free Space 0 8.85 × 10−12 C2 /N.m2
Permeability of Free Space µ0 4π × 10−7 T.m/A
Athmospheric Pressure 1 atm 1.0 × 105 M/m2
Bohr Radius a0 0.529 × 10−10 m

Table .1.1: Something


D

.2 Vector Identities

.2.1 Triple Products

A · (B × C) = B · (C × A) = C · (A × B) (.2.1)
A × (B × C) = B (A · C) − C (A · B) (.2.2)
cxxvi Constants & Important Equations
.2.2 Product Rules

∇ f g = f ∇g + g ∇ f
  
(.2.3)
∇ (A · B) = A × (∇ × B) + B × (∇ × A) + (A · ∇) B + (B · ∇) A (.2.4)
∇ · f A = f (∇ · A) + A · ∇ f
 
(.2.5)
∇ · (A × B) = B · (∇ × A) − A · (∇ × B) (.2.6)
∇ × f A = f (∇ × A) − A × ∇ f
 
(.2.7)
∇ × (A × B) = (B · ∇) A − (A · ∇) B + A (∇ · B) − B (∇ · A) (.2.8)

.2.3 Second Derivatives

.3 Commutators
∇ · (∇ × A) = 0
∇ × ∇f = 0


FT
∇ × (∇ × A) = ∇ (∇ · A) − ∇2 A
(.2.9)
(.2.10)
(.2.11)
RA
.3.1 Lie-algebra Relations

[A, A] = 0 (.3.1)
[A, B] = −[B, A] (.3.2)
[A, [B, C]] + [B, [C, A]] + [C, [A, B]] = 0 (.3.3)
D

.3.2 Canonical Commutator

[x, p] = i~ (.3.4)

.3.3 Kronecker Delta Function


(
0 if m , n;
δmn =
1 if m = n;
For a wave function Z
ψm (x)∗ ψn (x)dx = δmn (.3.5)

David S. Latchman ©2009


Linear Algebra cxxvii
.4 Linear Algebra

.4.1 Vectors

Vector Addition

The sum of two vectors is another vector

|αi + |βi = |γi (.4.1)

Commutative
|αi + |βi = |βi + |αi (.4.2)
Associative

FT
|αi + |βi + |γi = |αi + |βi + |γi
 
(.4.3)
Zero Vector
|αi + |0i = |αi (.4.4)
Inverse Vector
|αi + | − αi = |0i (.4.5)
RA
D

©2009 David S. Latchman


cxxviii Constants & Important Equations

FT
RA
D

David S. Latchman ©2009


Bibliography

[1] Wikipedia. Mean free path — wikipedia, the free encyclopedia, 2009. [Online;

FT
accessed 23-March-2009].

[2] John J. Brehm and William J. Mullin. Introduction to the Structure of Matter, chapter
12-3, page 594. Wiley, first edition, 1989.

[3] John J. Brehm and William J. Mullin. Introduction to the Structure of Matter, chapter
15-5, page 772. Wiley, first edition, 1989.

[4] John J. Brehm and William J. Mullin. Introduction to the Structure of Matter, chapter
RA
5-10, pages 283–287. Wiley, first edition, 1989.

[5] David J. Griffiths. Introduction to Quantum Mechanics, chapter 5.1.1, pages 203–205.
Prentice Hall, second edition, 2005.

[6] John J. Brehm and William J. Mullin. Introduction to the Structure of Matter, chapter
11-1, pages 539–540. Wiley, first edition, 1989.

[7] John J. Brehm and William J. Mullin. Introduction to the Structure of Matter, chapter
2-8, pages 114–116. Wiley, first edition, 1989.
D
Index

Angular Momentum, see Rotational Mo- Coupled Harmonic Oscillators, xx


tion Damped Motion, xix
Kinetic Energy, xviii
Bohr Model Potential Energy, xix

FT
Hydrogen Model, lv Simple Harmonic Motion Equation, xviii
Small Oscillations, xix
Celestial Mechanics, xxiv
GR9677 Q92, cxix
Circular Orbits, xxv
Total Energy, xviii
Escape Speed, xxiv
Kepler’s Laws, xxv Parallel Axis Theorem, see Rotational Mo-
Newton’s Law of Gravitation, xxiv tion
Orbits, xxv
RA
Potential Energy, xxiv Rolling Kinetic Energy, see Rotational Mo-
Circular Orbits, see Celestial Mechanics tion
Commutators, cxxvi Rotational Kinetic Energy, see Rotational
Canonical Commutators, cxxvi Motion
Kronecker Delta Function, cxxvi Rotational Motion, xxii
Lie-algebra Relations, cxxvi Angular Momentum, xxiii
Compton Effect, lviii Moment of Inertia, xxii
Counting Statistics, lxxi Parallel Axis Theorem, xxiii
Rolling Kinetic Energy, xxiii
Doppler Effect, xxii Rotational Kinetic Energy, xxii
Torque, xxiii
D

Franck-Hertz Experiment, lxi


Subject, xliv
Gravitation, see Celestial Mechanics
System of Particles, xxiv
Kepler’s Laws, see Celestial Mechanics
Torque, see Rotational Motion
Kronecker Delta Function, cxxvi
Vector Identities, cxxv
Linear Algebra, cxxvii
Product Rules, cxxvi
Vectors, cxxvii
Second Derivatives, cxxvi
Moment of Inertia, see Rotational Motion Triple Products, cxxv

Newton’s Law of Gravitation, see Celestial


Mechanics

Oscillatory Motion, xviii

Anda mungkin juga menyukai